Patho test 3 combo

Ace your homework & exams now with Quizwiz!

A nurse is providing discharge instruction of a client following a tracheostomy. Which of the following statements by the client indicates a need for further instruction? 1. "I need to inspect the stoma for signs of infection or skin irritation." 2. "I will clean the cannula with half-strength peroxide and rinse with saline." 3. "I can remove the old twill ties once the new ties are place." 4. "I should apply suction while inserting the catheter into my tracheostomy."

"I should apply suction while inserting the catheter into my tracheostomy." the client should apply suction only when withdrawing the catheter to prevent tracheal tissue trauma

A nurse is providing instruction to a client on how to use montelukast to treat chronic asthma. Which of the following statements indicates the clients understands the teaching? 1. "I will take this medication with each meal." 2. "I will take this medication during my asthma attacks." 3. "I will take this medication up to three times per day when I begin to wheeze." 4. "I will take this medication every evening, even when I do not have symptoms."

"I will take this medication every evening, even when I do not have symptoms." Montelukast is used for prophylaxis of asthma exacerbation and is taken on a daily basis in the evening. The client should take montelukast every day as maintenance therapy for asthma.

Hypercalcemia Manifestations:

* Bones Bradycardia, Kidney stones, bone pain, osteoporosis, pathologic fractures, anorexia, constipation

Hypernatremia ICF deficit Pathophysiology:

*(low on fluid w/ high Na+ too much Na+) Deficit of water in circulation as compared to sodium/particles. -Gain of Na+ or loss of H2O

Hypernatremia ICF deficit Etiologies:

*(low on fluid w/ high Na+ too much Na+) Enteral feedings, salt overuse, inability to respond to thirst, increased ADH , prolonged watery diarrhea (more h2O than salt)

Chemical Buffers include:

*Bicarb*, Phosphate buffer-from bones, Protein buffer-albumin/immunoglobulins

Hypornatremia ECF deficit (saline deficit) Etiologies: (hypovolemia)

*Volume issue (loss) Anything that causes loss of Na+ containing fluid such as diarrhea & vomiting. Renal excretion of Na+ ( low aldosterone = ADH = not peeing) Fluid into "third space"

Hypocalcemia Manifestations:

*muscle dysrhythmias, Trousseau's (limb spasms), Chvostek's (cheek), muscle twitching, cramping, tetany, tingling, diarrhea.

Hyponatremia ICF Excess Pathophysiology:

*too much water, not enough Na+ Fluid moves into cells, and they swell

Hyponatremia ECF deficit (saline deficit) Pathophysiology: (hypovolemia)

*volume issue (loss) Only extracellular fluid compartment is affected, since the fluid lost has the same osmolality as normal body fluids.

Hypernatremia ECF Excess (saline excess) Pathophysiology: (hypervolemia)

*volume issue (too much) Fluid gained has same osmolality as body fluids, so only the circulation and interstitial space is affected

Hypernatremia ECF excess (saline excess) Etiologies: (hypervolemia)

*volume issue (too much) Too much isotonic IV fluids Hyperaldosteronism (retaining Na+ & H20) Renal failure (not peeing) Heart failure (not circulating/backup) Corticosteroid (increase blood sugar which pulls more fluid)

Hypervolemia, Clients at risk include those with:

-Impaired renal function -Impaired cardiac function -Cirrhosis -An increased consumption of salt

Hyponetremia, Causes of

-kidney disease -GI losses -Adrenal insufficiency -Increased sweating -Use of diuretics -Decreased sodium intake

the PH range

0-14

Hypotonic Fluids

0.45 Normal Saline aka ½ NS

Aldosterone promotes which of the following? 1) Reabsorption of sodium. 2) Reabsorption of potassium. 3) Excretion of renin. 4) Excretion of water.

1

The plasma oncotic pressure does which of the following? 1) Pulls fluid back into the vascular space at the venous side of the capillary 2) Pushes fluid out of the vascular space at the venous side of the capillary 3) Pushes fluid out of the vascular space at the arterial side of the capillary 4) All of the above

1

Which of the following actions is important to observe when giving potassium chloride (KCl) 100 mEq intravenously (IV) to a client with severe hypokalemia? 1) Use an IV pump to ensure safe delivery. 2) Warm the solution to room temperature. 3) Give rapidly to counteract the hypokalemia. 4) Give undiluted as a rapid bolus.

1

Which of the following are symptoms of hypovolemia? 1) oliguria 2) weight gain 3) decreased pulse and increased BP 4) distended jugular veins

1

Which of the following meals is highest in calcium? 1) Salmon sandwich with spinach and a glass of milk. 2) Fried egg sandwich with nuts. 3) Taco with tomatoes 4) Ham sandwich with pickles .

1

Which of the following statements best describes electrolytes in extracellular and intracellular fluid? 1) There is a greater concentration of sodium in extracellular fluid and a greater concentration of potassium in intracellular fluid. 2) There is equal movement of sodium and potassium between intracellular and extracellular fluids. 3) There is a greater concentration of potassium in extracellular fluid and a greater concentration of sodium in intracellular fluid. 4) None of the above

1

The major cation regulating extracellular osmolality is: 1) Sodium 2) Potassium 3) Chloride 4) Magnesium

1 Cations are positively charged and anions are negatively charged. Sodium, potassium, and magnesium are all cations. Chloride is an anion. Sodium plays the most important part in regulating osmolality between and within the fluid compartments

An elderly patient has been ill with fever, muscle cramps, nausea and vomiting. His daughter calls the physician's office and asks what she should do. The nurse knows that these symptoms mostly likely represent what? 1) Hypovolemia 2) Hypocalcemia 3) Hypophosphatemia 4) Hypoaldosteronism

1 Fever, thirst and CNS system changes are signs of intracellular dehydration (more serious than extracellular). The N&V might have been the precipitating factor that led to the fluid volume loss resulting in hypovolemia. Fever also contributes to insensible water loss. The fact that he is elderly puts him at higher risk of fluid imbalance. Signs of hypocalcemia are bleeding, hypotension, palpitations, laryngospasm, double vision or tetany and seizures. Signs of hypophosphatemia are fatigue, weakness, bone pain, decreased cardiac function, confusion or seizures.

A patient developed a bowel obstruction after surgery and a nasogastric tube was inserted. The most common electrolyte imbalances secondary to gastric suctioning is what? 1) Hyponatremia / hypokalemia 2) Hyponatremia / hyperkalemia 3) Hypernatremia / hypocalcemia 4) Hypernatremia / hypokalemia

1 Patients with NG tubes are at risk for electrolyte disturbances especially if suction is used because electrolytes normally in the stomach secretions are removed from the body. Sodium, potassium, calcium, chloride, and magnesium deficiency are possible. Only NS should be used to irrigate an NG tube so it will not further alter the patient's fluid and electrolyte balance.

Which of the following statements about IV fluid replacement therapy for an infant with dehydration is true? 1) The infusion rate may be higher during the first 1-3 hours 2) The 1st half of the 24 hour total will be infused over 10 hours. 3) The infant is NPO for the first 48 hours. 4) Undiluted juices and cola may be given during IV replacement therapy.

1 Rehydration rates of infusion may be higher in the initial 1 to 3 hours than what is normally needed and then the rate will be slowed down to avoid fluid overload. If the dehydrated infant can drink fluids that can be an alternative way to provide fluids. Depending on the cause of dehydration, usually electrolyte solutions are used such as pedialyte or "New Oral Rehydration Solutions" as suggested by the World Health Organization for outbreaks of cholera and rotovirus rather than juice or cola.

A 15-year-old girl, the unrestrained passenger, in the front seat of a pickup truck was thrown from the vehicle when it hit a guard rail. She is admitted to the ER where she is evaluated by the ER physician. STAT lab work has been ordered and a student nurse asks why the doctor wants to have electrolytes done, "She's so young, and besides what abnormal labs could she have this early?" The best response would be: 1) "It is very possible that she will have an elevated potassium." 2) "We don't need to worry about what it will show; we'll just get it done." 3) "It is likely she will have an increased calcium due to her broken wrist." 4) "Electrolyte studies will indicate whether or not she had been drinking."

1 Severe trauma or tissue damage can release potassium into the extracellular compartment leading to a subsequent increase in serum potassium and the dangers that can cause. Blood alcohol level will indicate if she has been drinking. Even if she has fractured bones, her calcium level will probably not be affected at this point.

What are three things that have a osmotic pull?

1. Proteins (albumin) 2. Na+ 3. Blood Glucose

What are the three affects of aldosterone?

1. Retains Na+ 2. Excretes K+ 3. Vasocontriction

Renin>angiotensin>aldosterone system (RAAS) pg. 424

1. dehydration occurs 2. dehydration creates decreased blood volume 3. leads to decrease BP 4. Juxtaglomerular cells in the kidneys sense decrease BP 5. kidney releases Renin 6. Renin acts enzymatically on the angeotensinogen in the plasma that was released by the liver to convert angeotensin I 7. Angeotensin I goes to the lungs where ACE converts it to Angeotensin II 8. Angeotensin II produces vasoconstriction of the arterioles and adrenal cortex to release aldosterone> kidneys reabsorb Na+ and fluid follows the Na+ increases vascular volume and BP

normal levels of magnesium

1.3-2.5 mEq/L

What is the normal serum Mg+ levels?

1.5 - 2.5

Magnesium

1.5-2.5 mEq/L Acts as a catalyst for enzyme reactions. Regulates neuromuscular contractions, promotes normal functioning of the nervous and cardiovascular systems, aids in protein synthesis and sodium & potassium ion transportation.

Magnesium

1.5-2.5mg/dL 2nd most important cation of the ICF Essential for enzyme activities, neurochemical activities and muscular excitability Excreted through renal mechanism

Hyponetremia, Assessment and Diagnostic Findings

1.Labs indicate: -decreased serum and urine sodium -Decreased urine specific gravity 2.The following are normal values: -serum Na: 135-145 -Urine Na: 75-220mEq/day -Urine specific gravity: 1.016-1.022

Extracellular fluid

1/3

What is the normal serum Na+ level?

135-145 mEq/L

Normal level of Na+

135-145 mEq/L. Found in the ECF

The majority of the body's water is contained in which of the following fluid compartments? 1) interstitial 2) intracellular 3) extracellular 4) intravascular

2

Which of the following interventions would be the best choice to monitor fluid and electrolyte balance? 1) assess if client is voiding 2) assess daily weight 3) evaluate daily urine specimens 4) check daily sodium levels

2

Which of the following is the normal level of potassium in the blood? 1) 1.5 to 4.0 2) 3.5 - 5.0 3) 5.0-7.5 4) none of the above

2

A patient has been admitted with a potassium of 5.8 mEq/L; the nurse anticipates which of the following action? 1) Administration of glucose IV and insulin sub-q. 2) Administration of glucose / insulin / diuretics IV. 3) Administration of IV potassium diluted in normal saline. 4) Administration of synthetic aldosterone.

2 Normal Potassium range is between 3.5 to 5.0 mEq/L. Increased potassium can lead to serious cardiac arrhythmias and actions must be taken to eliminate excess potassium. In non-emergency situations, hyperkalemia can be treated with a low potassium diet. Insulin injections are used to treat hyperkalemia in emergency situations. It provokes the uptake of potassium ions by cells, decreasing potassium ion concentration in the blood. Kayexalate is a resin given orally or rectally that can help bind potassium in the intestines for excretion by the feces.

Two-thirds of total body fluid is located in which compartment? 1) Interstitial 2) Intracellular 3) Intravascular 4) Extracellular

2 Approximately 70% of body fluid is intracellular and 30% is extracellular. In order for the cells of the body to function normally, this balance must be maintained.

During the administration of IV fluids, which of the following findings should prompt further action? 1) Increased urinary output. 2) Crackles heard during pulmonary assessment. 3) Lack of thirst. 4) Abdominal cramping.

2 Crackles heard upon respiratory assessment could indicate fluid retention or overload (pulmonary edema).The physician needs to be notified of this development.

For the patient in hemorrhagic (hypovolemic) shock, the most important nursing action is to: 1) Administer replacement electrolytes orally or by IV. 2) Administer adequate fluid / blood volume IV. 3) Place the patient on cardiac monitor to observe for signs of electrolyte abnormalities. 4) Monitor urinary output as a measure of fluid status.

2 Fluid replacement is most important to maintain function of the cells for the patient in hypovolemic shock. The patient should be maintained NPO in case surgery, including anesthesia, is required. Remember ABCs. Fluid replacement is part of C.

A patient is being discharged following an acute myocardial infarction and subsequent heart failure. His discharge medications include: Lasix 40 mg daily, Lanoxin 0.25 mg daily, and the ACE inhibitor, Capoten (captopril) 50 mg TID. Prior to sending him home, the nurse should call the physician regarding: 1) The time of day the medications are to be taken. 2) The lack of a potassium supplement order. 3) The lack of fluid restriction order. 4) The lack of chloride and magnesium supplement orders.

2 He is on a diuretic (Lasix) and this would alert the nurse to the idea that he should have a potassium replacement ordered. However, the Capoten is potassium sparing and could increase serum potassium levels. So it would be best to check with the MD about these orders and if supplementation is needed!

Phosphorus

2.5 - 2.5 mg/L Helps maintain bones and teeth along with calcium Regulated by kidneys, parathyroid hormones and Vitamin D Calcium and phosphorus are inversely related. If one rises, the other one falls

Normal levels of Phosphorus

2.5 to 4.5 mg/dl

Intracellular fluid makes up this amount of body weight

2/3

NaHCO3 (sodium Bicarb) & H2CO3 (carbonic acid) are at what ratio?

20:1- 20 bases to 1 acid

HCO3 normal value

22-26

homiostatic osmilarity in the body

270-300 m osm/kg h2o

A client is hyponatremic and the cause is thought to be fluid overload. When this is the etiology, what would be the intervention of choice? 1) restrict sodium intake 2) do nothing and allow it to resolve itself 3) restrict oral fluids 4) force fluids

3

A patient is receiving a loop diuretic. The nurse should be alert to which of the following symptoms? 1) Restlessness and agitation 2) Paresthesias and irritability 3) Weak, irregular pulse, cardiac arrhythmias, and muscle weakness 4) Increased blood pressure and muscle spasms

3

Which of the following diets contain the most sodium? 1) Roast beef sandwich with mustard and an apple 2) Turkey sandwich with mashed potatoes 3) Bologna sandwich with pickles and potato chips 4) Veggie wrap with pears

3

When evaluating the effectiveness of IV fluid re-hydration for severe dehydration, which of the following is the best indicator? 1) Weight gain 2) Skin turgor 3) Increased urinary output 4) Increased temperature

3 For a person with more acute or severe dehydration, you would expect to see increase in urine output and decrease in urine osmolarity with re-hydration. Other signs re-hyrdration include moist mucous membranes, absence of tongue furrows, absence of orthostatic hypotension. Careful monitoring of I&O can help assess the patient's progress towards the goal of re-hydration.

The preferred route for potassium replacement is: 1) IV push 2) IV Piggyback 3) Oral 4) Continuous drip

3 Low potassium is most often seen in patients who have polyuria such as in diabetes or those on diuretics. An oral potassium supplement is usually prescribed and is the preferred route if tolerated. IV potassium should only be used with extreme caution. IV Boluses of potassium (IV push) may cause severe and potentially fatal heart rhythm disturbances, so careful cardiac monitoring is required during IV drug administration. IV piggyback doses of potassium can be given slowly if correction is needed quickly. Potassium can be added to IV solutions for maintenance or slower replenishment of the electrolyte.

Which of the following laboratory results places the patient at greatest risk for dysrhythmias? 1) Na+ 148 mEq/L 2) Ca++ 8.9 mg/dL 3) K+ 6.6 mEq/L 4) Na+ 133 mEq/L

3 Watch out for that Potassium! Normal is 3.5-5.0 (memorize this!!!!) Hyperkalemia can lead to cardiac dysrhythmias.

Normal level of potassium K+

3.5-5 mg/dl

What is the normal serum K+ levels?

3.5-5.0

interstitail fluid

3/4 of extracellular fluid found in Lymph, spinal cord, blood plasma, plural cavity

Co2 normal value

35-45

The active process by which sodium moves out of the cell and potassium moves into the cell is called: 1) filtration. 2) osmosis. 3) diffusion. 4) active transport.

4

Which of the following are symptoms of hypocalcemia (low calicum)? 1) Positive Chvostek's and Trousseau signs and seizures 2) Prolonged QT interval 3) Numbness and tingling of the hands and muscle cramps 4) All of the above

4

Which of the following is responsible for the oncotic pressure in the blood stream? 1) Antidiuretic hormone 2) Blood urea nitrogen 3) Creatinine 4) Albumin

4

An example of an isotonic solution is: 1) D5 1/2 Normal Saline 2) D5 Lactated Ringers 3) 1/2 Normal Saline 4) Normal Saline

4 An isotonic solution has an osmotic pressure equal to plasma (0.9 NS). Dextrose Solutions (D5 1/2 NS, D5 NS, or D5 LR) are hypertonic and anything less than 0.9 NS is hypotonic. Ringer's solution without dextrose is isotonic. Hypertonic fluids also include fluids with 3-5% NS or high destrose solutions such as D10 or D50 in water.

The IV fluid should be used to prime tubing for administration of blood is: 1) D5 1/2 NS 2) D5 LR 3) 1/2 NS 4) NS

4 Only isotonic saline (0.9%) is recommended for use with blood components. Other isotonic electrolyte solutions that do not contain calcium may be used according to hospital policy. Other commonly used intravenous solutions will cause varying degrees of difficulty when mixed with red cells. For example, 5% dextrose in water will hemolyze red cells. Intravenous solutions containing calcium, such as Lactated Ringer's solution, can cause clots to form in blood. Prior to blood transfusion, completely flush incompatible intravenous solutions and drugs from the blood administration set with isotonic saline.

Total body weight that comes from water

60%

neutral PH

7

functioning PH level in the blood

7.35-7.45

Normal levels of Ca+

8.5-11 mg/dl

Calcium

8.6 - 10.5 mg/dl Needed for cell membrane integrity & structure, adequate cardiac impulse conduction, blood coagulation, bone growth and integrity, muscle relaxation Regulated through the actions of the parathyroid and thyroid glands 99% calcium found in bones and teeth

PO2 normal value

80-100

What is the normal serum Ca+ levels?

9-11 mg/dl

O2 sat normal value

95-100%

Chloride

96 - 106mEq/L Primary ECF anion Fluctuations in chloride levels usually parallel sodium Maintained by dietary intake and renal excretion and reabsorption Regulated through the kidneys Gastric mucosa cells need chloride to produce HCL acid

Chloride

98-108 mEq/L Located outside of cells. Helps maintain osmotic pressure (water-pulling pressure). Gastric mucosal cells need chloride to produce hydrochloric acid, which breaks down food into absorbable components.

Hypocalcemia Pathophysiology:

< 9 Low ionized calcium results in ↑ neuromuscular irritability

Hypophosphotemia

<2.5 mg/dl

Metabolic acidosis

<22 mEq/ L , Bicarb deficit, loss through diarrhea or renal dysfunction. occasionally is caused by the accumulation of lactic acid or ketones. Failure of kidneys to secrete H+ ions. Diabetic ketoacidosis. High protein no carb diets can cause this. PH will be < 7.35 CO2 will be between 35-45.

Hypochloremia

<96 mEq/L Causes: Salt restricted diets GI drainage Severe vomiting and diarrhea

Hypercalcemia

>11 High serum ionized Ca2+ leads to ↓ neuromuscular irritability

Hypermagnesium

>2.5mg/dl in ECF

Hyperphosphotemia

>4.5mg/dl

A nurse receives prescriptions from the provider to perform nasopharyngeal suctioning for each of the following clients. The nurse should clarify the provider's prescription for which of the following clients? 1. A client who has a closed-head injury and is lethargic 2. A client who has a fractured femur and reports severe pain 3. A client who has a ruptured appendix and a temperature of 39 C (102.2 F) 4. A client who has emphysema and respiration's of 36/min

A client who has a closed-head injury and is lethargic A recent head injury is a contraindication for nasopharyngeal suctioning because suctioning can increase intracranial pressure.

What is the appropriate term for the "thick filaments of myosin that constitute a central dark band"? A. A band B. I band C. Z line D. M line

A. A band

Which disorder can be the cause of pulmonary fibrosis? Select all that apply. A. Acute respiratory distress syndrome (ARDS) B.Tuberculosis C.Inhalation of excessive amounts of coal dust D.Rheumatoid arthritis E.Upper respiratory infections

A. Acute respiratory distress syndrome (ARDS) B.Tuberculosis C.Inhalation of excessive amounts of coal dust D.Rheumatoid arthritis

A 23-year-old patient presents with severe chest pain that worsens with respiratory movements and with lying down. Which is the most appropriate intervention for this condition? A. Administer analgesics B. Percutaneous coronary intervention C. Total bedrest D. Give beta blockers

A. Administer analgesics

Which statement regarding autoregulation is TRUE? Select all that apply. A. Blood vessels regulate their own blood flow. B..In coronary circulation, pressure is between 60 mmHg and 180 mmHg. C.Mechanism is well documented and is related to the sympathetic system. D. Autoregulation ensures constant coronary blood flow. E. It is proposed that this process originates in vascular smooth muscles of the arterioles.

A. Blood vessels regulate their own blood flow. B..In coronary circulation, pressure is between 60 mmHg and 180 mmHg. D. Autoregulation ensures constant coronary blood flow. E. It is proposed that this process originates in vascular smooth muscles of the arterioles.

Which enzyme is released by damaged myocardial muscle tissue? Select all that apply. A. CK-MB B. LDH C. AST D. Troponin I E. ALT

A. CK-MB B. LDH D. Troponin I

Which medical condition places a client at high risk for developing chronic upper airway obstruction? Select all that apply. A. Congenital malformation B. Infection C. Angioedema D. Facial trauma E. Subglottic stenosis

A. Congenital malformation E. Subglottic stenosis

Which condition can cause of hypercapnia? Select all that apply. A. Disease of the medulla B. Large airway obstruction C. Increased respiratory drive D. Thoracic cage abnormalities E. Depression of the respiratory center

A. Disease of the medulla B. Large airway obstruction D. Thoracic cage abnormalities E. Depression of the respiratory center

An adult is diagnosed with a chronic cough resulting from a viral infection. What nursing intervention is appropriate for this client? Select all that apply. A. Encourage the client to return for treatment if the cough has not resolved in 8 weeks B.Assess for a history of gastroesophageal reflux disease (GERD) C. Educate the client to strategies to prevent upper respiratory tract infections D. Assess for a history of allergic rhinitis E. Determine whether the client is a smoker

A. Encourage the client to return for treatment if the cough has not resolved in 8 weeks B.Assess for a history of gastroesophageal reflux disease (GERD) D. Assess for a history of allergic rhinitis

Which statement regarding chronic bronchitis is TRUE? A. Hypersecretion of mucus is a characteristic. B. It features a chronic productive cough that lasts at least 6 months. C. Symptoms occur for at least 5 consecutive years. D. Is one of the few respiratory diseases not exacerbated by smoking

A. Hypersecretion of mucus is a characteristic.

A patient is diagnosed with inflammation of the endocardium by staphylococcus aureus. Which is the most likely medical diagnosis? A. Infective endocarditis B. Pericardial effusion C. Chronic pericarditis D. Cardiomyopathy

A. Infective endocarditis

Which equation is used to determine mean arterial pressure (MAP)? A. MAP = diastolic + 1/3 (systolic - diastolic) B. MAP = VP C. MAP = (P1 - P2) /resistance D. MAP = pressure ¿ radius/wall thickness

A. MAP = diastolic + 1/3 (systolic - diastolic)

A patient has impaired blood flow from the left atrium to the left ventricle. Which valvular problems does this describe? A. Mitral stenosis B. Mitral regurgitation C. Aortic stenosis D. Aortic regurgitation

A. Mitral stenosis

Which of the following represents the measure of time from the onset of atrial activation to the onset of ventricular activation? A. PR interval B. QRS complex C. ST interval D. QT interval

A. PR interval

A young woman presents with pallor, numbness, and a sensation of cold temperature of her digits. Which condition is the most likely cause for these symptoms? A. Raynaud phenomenon B. Thromboangiitis obliterans C. Peripheral vascular disease D. Varicose veins

A. Raynaud phenomenon

Which items are related in the Frank-Starling law of the heart? A. Resting sarcomere length to tension generation B. Resting sarcomere length to end-diastolic volume C.Tension generation and left ventricular pressure D. Tension generation and diastolic filling pressures

A. Resting sarcomere length to tension generation

Identify the pathway that deoxygenated blood returning through the superior and inferior venae cavae takes to the lung of re-oxygenation? A begins the pathway and E ends it. A. Right atrium B. Tricuspid valve C. Right ventricle D. Semiluar valve E. Pulmonary artery

A. Right atrium B. Tricuspid valve C. Right ventricle D. Semiluar valve E. Pulmonary artery

Which client has an increased risk for developing a pulmonary embolus (PE)? Select all that apply. A. The adult in traction for a fractured femur B.The woman taking birth control pills C.The child with exercise-induced asthma D. The teenager receiving intravenous chemotherapy E. The man with a prothrombin gene mutation

A. The adult in traction for a fractured femur B.The woman taking birth control pills D. The teenager receiving intravenous chemotherapy E. The man with a prothrombin gene mutation

What factor will be influential in the development of aspiration pneumonia? Select all that apply. A. The amount of aspirate B. pH of the material aspirated C. Size of the particles aspirated D. Protein content of the aspirate E. Bacterial content present in the aspirate

A. The amount of aspirate B. pH of the material aspirated C. Size of the particles aspirated E. Bacterial content present in the aspirate

Which items are components of the heart's anatomy? Select all that apply. A. Two atria B. Two atrioventricular valves C. Two ventricles D. Two-layer heart wall E. Two semilunar valves

A. Two atria B. Two atrioventricular valves C. Two ventricles E. Two semilunar valves

Which are steps in the process of gas exchange? Select all that apply. A. Ventilation B. Sighing C. Diffusion D. Perfusion E. Rebreathing

A. Ventilation C. Diffusion D. Perfusion

The respiratory center is correctly identified as: Select all that apply. A. being located in the brain stem. B. affecting the respiratory muscles' ability to contract and relax. C. containing one major type of neuron. D. transmitting impulses that bring about the physical act of breathing. E. being influenced by emotions and pain.

A. being located in the brain stem. B. affecting the respiratory muscles' ability to contract and relax. D. transmitting impulses that bring about the physical act of breathing. E. being influenced by emotions and pain.

When discussing common clinical manifestations of hypokalemia (low serum potassium), it is correct to state that: Select all that apply. A. carbohydrate metabolism is affected due to decreased insulin secretion. B.renal function is impaired. C. neuromuscular excitability is decreased. D.skeletal muscle contractility is increased. E.smooth muscle tone is increased.

A. carbohydrate metabolism is affected due to decreased insulin secretion. B.renal function is impaired. C. neuromuscular excitability is decreased.

Which statement appropriately defines hypoventilation? Hypoventilation is: A. inadequate alveolar ventilation in relation to metabolic demands. B. rapid breathing with little to no expiratory pause. C. difficulty breathing due to body position. D. the feeling of not getting enough air.

A. inadequate alveolar ventilation in relation to metabolic demands.

The only action that would not cause a release on renin is the: A. increase in blood pressure at the renal artery. B. decrease in the amount of sodium delivered to the kidney. C. beta-adrenergic stimuli. D. low potassium concentration.

A. increase in blood pressure at the renal artery.

A common cause of the increased filtration of fluid from capillaries and lymph into surrounding tissues (edema) includes: Select all that apply. A. increased hydrostatic pressure. B. decreased plasma oncotic pressure. C. increased capillary membrane permeability. D. lymphatic obstruction. E.increased barometric pressure.

A. increased hydrostatic pressure. B. decreased plasma oncotic pressure. C. increased capillary membrane permeability. D. lymphatic obstruction.

Which is a function of the pericardium? The pericardium: Select all that apply. A. prevents displacement of the heart during gravitational acceleration/deceleration. B.acts as a physical barrier that protects the heart against infection and inflammation. C. contains pain receptors and mechanoreceptors that affect blood pressure. D.provides a continuous lining of the arteries, veins, and capillaries of the body. E.brings about the force required to normalize blood pressure.

A. prevents displacement of the heart during gravitational acceleration/deceleration B.acts as a physical barrier that protects the heart against infection and inflammation. C. contains pain receptors and mechanoreceptors that affect blood pressure. E.brings about the force required to normalize blood pressure.

Which drugs have a major effect in decreasing the strength of cardiac contraction? A.Calcium channel blockers B. Narcotics (morphine) C. Adenosine D. Aspirin

A.Calcium channel blockers

Which characteristic is associated with hypertension? Select all that apply. A. Family history positive for hypertension B. Asian race C. High dietary sodium D. Glucose intolerance E. Obesity

A.Family history positive for hypertension C. High dietary sodium D. Glucose intolerance E. Obesity

When discussing aldosterone, it is correct to state that: A: it is a hormone that is secreted when sodium levels are depressed. B: it is a hormone that is secreted when potassium levels are depressed. C: its action is to decrease the retention of sodium. D: its action is to decrease the secretion of potassium.

A: it is a hormone that is secreted when sodium levels are depressed.

A patient has deep and rapid respirations. Laboratory tests reveal decreased pH and bicarbonate. This patient is experiencing: A: metabolic acidosis. B: metabolic alkalosis. C: respiratory acidosis. D: respiratory alkalosis.

A: metabolic acidosis.

When discussing body fluid, it is correct to state that: A: two-thirds of the body's water is intracellular. B: one-fourth of the body's fluid is extracellular. C: the two main extracellular compartments are interstitial and intracellular. D: standard total body water is 40% of body weight.

A: two-thirds of the body's water is intracellular.

Diabetes insipidus (DI) is a condition that is caused by a deficiency of or a decreased renal response to _____, also known as vasopressin.

ADH

CO2 levels during respiratory acidosis

Above 45 mm Hg *hypercapnea*

What moves the opposite of Mg+

Acetylcholine (ACh)

fluid can be determined by body compensation

Adipose tissue displaces water Muscle tissues hold on to water

Medical Management of Hypophosphatemia

Administer Fleets Phospho Soda Administer IV phosphorus if: The GI tract is not functioning The serum level is dangerously low - below 1mg/dl

Hypernatremia, Medical Mangement

Administer hypotonic IV solutions to gradually lower the serum sodium level eg: 0.3% sodium chloride or D5W Diuretics may be given to lower the sodium level A rapid reduction in the serum sodium level may cause cerebral edema

What are body fluids affected by?

Age, Gender, & Percentage of body fat. Babies: 75-80% of body weight is fluid. Young Adults: 60% of body weight is fluid. Older Adults: 45-50% of body weight is fluid, decreased thirst response. *very young are very old are more susceptible to fluid imbalances. Males: Greater % of body weight is H2O. Muscle contains the most H2O *Obese individuals carry a lower % of body weight as H2O.

What does renin produce?

Angiotensin I ---> Angiotensin II ---> Aldosterone

Manifestation of Hypocalcemia

Arrhythmia,trousseaus, Chvostek, nerves more easily fire, convulsions, tetany and spasms, intestinal cramping and hyperactive bowel sounds, labored breathing

A nurse is caring for a client in acute respiratory failure who is receiving mechanical ventilation. Which of the following assessments is the priority for the nurse to use to evaluate the effectiveness of the mechanical ventilation? 1. blood pressure 2. capillary refill 3. arterial blood gases 4. heart rate

Arterial blood gases when using the airway, breathing, circulation approach the client care, the nurse should place priority on evaluating arterial blood gases to determine serum oxygen saturation and acid-base balance.

Hypervolemia, Nursing Management of

Assess vs and hemodynamic status Monitor respiratory status Maintain strict I&O Monitor breath sounds, weight & edema Maintain IV access, Elevate HOB Administer oxygen as ordered Provide mouth and skin care

Which is the MOST common complication of an AMI? A. Dressler syndrome B. Arrhythmia C. Pericarditis D. Heart failure

B. Arrhythmia

Which term describes the occlusion of a blood vessel from a bolus of circulating matter in the bloodstream? A. Thrombus B. Embolus C. Thrombophlebitis D. Foam cell

B. Embolus

A patient is being transferred from another medical facility with a diagnosis of acute myocardial infarction (AMI). Which symptom is typically observed with the presences of an AMI? Select all that apply. A. Cough B. Fever C. Diaphoresis D. Substernal pain E. Cool, clammy skin

B. Fever C. Diaphoresis D. Substernal pain E. Cool, clammy skin

Which of the following would shift the oxyhemoglobin dissociation curve to the left? A. Acidosis B. Hypocapnia C. Increased H+ ion D. Increased metabolic activity

B. Hypocapnia

Which pathology most commonly results in pulmonary edema? A. The inhalation of organic particles causing an inflammatory response B. Increasing pressure in the left chambers of the heart C. Inflammatory obstruction of small airways D. The permanent enlargement of acini

B. Increasing pressure in the left chambers of the heart

What information will the nurse include when educating a client regarding their diagnosis of empyema? Select all that apply. A. Symptoms include bloody sputum. B. It is the presence of pus in the pleural space. C. A complication of respiratory infection often occurs. D. Diagnosis is made by thoracentesis and sputum culture. E. Breath sounds are increased directly over affected area.

B. It is the presence of pus in the pleural space. C. A complication of respiratory infection often occurs. D. Diagnosis is made by thoracentesis and sputum culture.

A patient has a loud pansystolic murmur that radiates to the back and axilla. Which valvular abnormality is this describing? A. Mitral stenosis B. Mitral regurgitation C. Aortic stenosis D. Aortic regurgitation

B. Mitral regurgitation

Which structure is a part of the upper airway? A. Trachea B. Nasopharynx C. Larynx D. Bronchi

B. Nasopharynx

Which describes the cardio-inhibitory center? A. Sympathetic excitatory neurons B. Parasympathetic excitatory neurons C. Bainbridge reflex D. Baroreceptor reflex

B. Parasympathetic excitatory neurons

A patient is born with a α-antitrypsin deficiency. Which of the following conditions will most likely manifest? A. Asthma B. Primary emphysema C. Pulmonary fibrosis D. ARDS

B. Primary emphysema

A patient presents with chest pain that occurs at rest. Abnormal vasospasm of the coronary arteries is detected. Which is the most likely cause of the chest pain? A. Stable angina B. Prinzmetal angina C. Silent ischemia D. Angina pectoris

B. Prinzmetal angina

What is the process called where carbon dioxide (CO2) is exchanged for oxygen? A. Ventilation B. Respiration C. Circulation D. Alveolar ventilation

B. Respiration

Which structures are considered part of the acinus? Select all that apply. A. Trachea B. Respiratory bronchioles C. Alveolar ducts D. Alveoli E. Nare

B. Respiratory bronchioles C. Alveolar ducts D. Alveoli

Which statement regarding hypocapnia is TRUE? Select all that apply. A. Severe cases sometimes result in disorientation B. Should be assessed for in clients experiencing severe anxiety C. Caused by inadequate alveolar ventilation D. Results in respiratory alkalosis E. The condition confirmed by a PaCO2 less than 36 mm Hg

B. Should be assessed for in clients experiencing severe anxiety D. Results in respiratory alkalosis E. The condition confirmed by a PaCO2 less than 36 mm Hg

Which part of the heart is responsible for electrical impulse stimulation? A. Atrioventricular (AV) node B. Sinoatrial (SA) node C. Bundle of His D. Right bundle branch

B. Sinoatrial (SA) node

What is the effect of angiotensin II? A. Vasodilation B. Vasoconstriction C. Inhibition of aldosterone D. Excretion of sodium at the kidney

B. Vasoconstriction

A patient experiences spinal cord damage following a motorcycle accident. Which type of shock would most likely result? A. Hypovolemic B. Vasogenic C. Anaphylactic D. Septic

B. Vasogenic

Which conditions are associated with dyslipidemia? A. High-density lipoprotein (HDL) - triglycerides and fat packages B. Very low-density lipoproteins (VLDL) - triglycerides and proteins C. Low-density lipoproteins (LDL) - phospholipids and proteins D. Chylomicrons - triglycerides and phospholipids

B. Very low-density lipoproteins (VLDL) - triglycerides and proteins

What is the best method for minimizing the spread of tuberculosis from person to person? A. Safe sex practices B. Wearing a face mask C. Good hand washing technique D. Frequent rinsing of the nasal mucosa

B. Wearing a face mask

Which statement regarding the thoracic duct is TRUE? The thoracic duct: A. is a major source of venous return to the heart. B. receives lymph from most of the body. C. collects lymph from the right arm, head, and thorax. D. dumps blood into the right atrium.

B. receives lymph from most of the body.

Which statement regarding arteriosclerosis is TRUE? Arteriosclerosis is: A. an acute process of heart muscle degeneration. B. thickening and hardening of the vessel wall. C. plaque caused by neutrophils. D. fatty streaks and foam cells that are identical

B. thickening and hardening of the vessel wall.

Which artery travels in the coronary sulcus between the left atrium and the left ventricle? A. Left anterior descending B.Circumflex C. Right coronary D. Left coronary

B.Circumflex

A patient has been diagnosed with a pericardial effusion resulting in the presence of frank blood. Which condition is associated with a bloody effusion? A. Tuberculosis B.Coagulation defect C. Neoplasm D. Left heart failure

B.Coagulation defect

When discussing causes of hypernatremia (elevated serum sodium), it is correct to include: Select all that apply. A. excessive free water intake. B. inappropriate administration of hypertonic saline solution. C. oversecretion of the hormone aldosterone. D. Cushing syndrome. E.ingesting large amounts of dietary sodium.

B.inappropriate administration of hypertonic saline solution. C. oversecretion of the hormone aldosterone. D.Cushing syndrome.

Which of the following is a TRUE statement regarding hyperkalemia (elevated serum potassium)? A: Dietary excesses are commonly the cause. B: It often occurs in acidosis. C: There is an increased in the renal secretion of potassium. D: Increased aldosterone levels are noted.

B: It often occurs in acidosis.

purpose of phosphorus

Bone and ATP formation, glucose, fat and protein metabolism, buffer for acid, RBC, WBC platelet function

purpose of Ca+

Bone strength and stability, membrane and potentials and excitability, contraction of all muscle types, blood clotting, second messenger in many hormone and neurotransmitter pathways

What part of the body does Ca+ effect?

Bones & Muscles

respiratory sensing center is located?

Brain stem in the medula obligata. Very sensitive to PH

Treatment for respiratory alkalosis

Bring back to lower altitudes, breath into paper bag, if unable to breath in paper bag person passes out, IV chloride containing solutions, Cl ions replace lost Bicarb ions.

Which statement regarding bronchiolitis is TRUE? A. In adults, it is usually caused by a virus. B. It is most common in chronic bronchitis. C. An inflammatory obstruction of small airways occurs. D. Bacterial infection is a causative agent.

C. An inflammatory obstruction of small airways occurs.

Which complication occurs secondary to hypertension? A. Cardiovascular muscle atrophy B. Hypoglycemia C. Congestive heart failure D. Decreased demand for coronary perfusion

C. Congestive heart failure

A patient has symmetrically dilated airways. Which type of bronchial dilation is this? A. Saccular B. Varicose C. Cylindrical D. Bronchiectasis

C. Cylindrical

Which muscle has a major role in respiration? Select all that apply. A. Sternocleidomastoid B. Trapezius C. External intercostal D. Diaphragm E. Scalene

C. External intercostal D. Diaphragm

What two factors determine cardiac output? A. Parasympathetic and sympathetic activity B. Preload and afterload C. Heart rate and stroke volume D. Right and left atrial pressure

C. Heart rate and stroke volume

Which statement regarding cellular injury of the myocardium is correct? A. Cardiac cells can withstand ischemic conditions for about 40 minutes. B. ECG changes are visible after approximately 120 seconds. C. Myocardial cells remain viable if blood flow returns within 20 minutes. D. After 20 seconds of decreased blood flow, myocardial cells become cooler.

C. Myocardial cells remain viable if blood flow returns within 20 minutes.

A client experiences an increased pressure in the chest cavity with resulting collapse of the right lung and is diagnosed with a secondary pneumothorax. This diagnosis is based on what criterion? A. Tracheal deviation away from the affected lung is present. B. The exact cause of the pathology is not known. C. The client experienced some type of chest trauma resulting in the symptoms. D. The rupture of blebs on the visceral pleura resulted in the symptoms.

C. The client experienced some type of chest trauma resulting in the symptoms.

Which statement regarding the lower airway is accurate? A. The trachea bifurcates at the larynx. B. The trachea has no supportive structure. C. The right and left main bronchi enter the lungs at the hila. D. The goblet cells are air-filled cells.

C. The right and left main bronchi enter the lungs at the hila.

Which statement regarding cigarette smoke is correct? Cigarette smoking produces: A. decreased LDL . B. increased HDL . C. increased thrombotic state. D. decreased inflammatory state.

C. increased thrombotic state.

A condition in which blood has pooled, producing distended and palpable vessels, is referred to as a: A. thrombus. B. venous stasis ulcers. C. varicose veins. D. deep vein thrombosis.

C. varicose veins.

When discussing hyponatremia (low serum sodium), it is correct to state that it is generally: A: a result of inadequate sodium intake. B: a result of a decrease in total body water (TBW). C: an outcome of serious burns, vomiting, or diarrhea. D: an outcome of serum sodium levels dropping below 145 mEq/L.

C: an outcome of serious burns, vomiting, or diarrhea.

Which of the following statements is TRUE? When discussing body fluid movement, it is correct to state that: A: potassium is the most abundant extracellular fluid (ECF) ion. B: sodium maintains the osmotic balance of the intracellular fluid (ICF) space. C: water moves freely across membranes. D: aquaporins block water movement across the membrane.

C: water moves freely across membranes.

What does the Parathyroid regulate?

Ca+

Hypocalcemia

Ca+ serum levels < 8.5 mg/dl

Hypercalcemia

Ca+ serum levels > 11 mg/dl

Serum _____ levels are directly or indirectly regulated by parathyroid hormone (PTH) and vitamin D.

Calcium

Medical Management of Hypocalcemia

Calcium supplement IV administration of Calcium in D5W via slow infusion Calcium gluconate, Calcium chloride Calcium gluceptate Vitamin D therapy to increase calcium absorption from the GI tract

Ca+

Calcium. 8.9 - 10.1 mg/dL Located outside of cell. The major cation involved in the structure and function of bones and teeth. Needed to: stabilize the cell membrane and reduce its permeability to sodium, transmit nerve impulses, contract muscles, coagulate blood, and form bone and teeth.

The most serious effect of hyperkalemia is on _____ function.

Cardiac

What part of the body does K+ effect?

Cardiac

Localized Edema

Caused from injury/tissue damage- sprains and strains Dangerous if occurs in larynx and brain, and lungs

Hypomagnesemia

Causes: Alcohol withdrawal - most common cause Prolonged inadequate dietary intake of magnesium Loss from the GI tract by NG suction, diarrhea, or fistulas Problems with the distal small bowel

what produces CO2?

Cellular metabolism

first line of defense against PH change in the body

Chemical buffer system

Etiology of Hypokalemia

Chronic kidney disorders, adrenal tumors, increased aldosterone (hyperaldosteronism), decreased intake, Emisis(vomiting), GI sux, Diuretics, shift in ph >7.5 (metabolic alkalosis), insulin shifts K+ back into to cell, asthma medications

Hypermagnesemia

Clinical Manifestations: Lethargy, drowsiness, coma Decreased deep tendon reflexes Muscle weakness Paralysis Depressed respirations Hypotension AV block Cardiac arrest

Hyperphosphatemia

Clinical Manifestations: tetany ( involuntary contraction of muscles) Serum level > 4.5mg/dl Causes: Renal failure Chemotherapy DKA

Intracellular imbalances are what kind of issue?

Concentration *One is too high (excess) or too low (deficit)

Osmolality

Concentration of particles in fluid

Respiratory Alkalosis can be caused by......

Conditions that stimulate respitory center,*Oxygen deficiency at high altitudes*, *Pumonary disease* and CHF, *acute anxiety*, fever, anemia-lack of oxygen carrying hemoglobin, early aspirin intoxication, Chirrosis, gram negative sepsis

A nurse is assessing a client who has a chest tube in place following thoracic surgery. Which of the following findings indicated a need for intervention? 1. Fluctuation of drainage in the tubing with inspiration 2. continuous bubbling in the water seal chamber 3. drainage of 75mL in the first hour after surgery 4. several small, dark-red blood clots in the tubing

Continuous bubbling in the water seal chamber continuous bubbling in the water seal chamber suggests an air leak

Which classic breathing pattern is generally identified with Cheyne-Stokes respirations? A. A lack of expiratory pause B. Presence of occasional sighs C. Rapid cycle of inspirations and expirations D. Alternating periods of deep and shallow breathing

D. Alternating periods of deep and shallow breathing

Which receptor monitors pH, PaCO2, and PaO2 of arterial blood? A.Stretch receptors B. Irritant receptor C. J receptors D. Chemoreceptors

D. Chemoreceptors

A patient experiences an elevated systolic pressure accompanied by normal diastolic pressure (below 90 mmHg). Which is the correct term for this condition? A. Primary hypertension B. Secondary hypertension C. Tertiary hypertension D. Isolated systolic hypertension

D. Isolated systolic hypertension

A patient has rheumatic fever. Which valve is most commonly affected? A. Aortic B. Tricuspid C. Pulmonic D. Mitral

D. Mitral

Which hormone is released from heart tissue in response to increases in blood volume? A. Epinephrine B. Norepinephrine C. Thyroid hormone D. Natriuretic peptide

D. Natriuretic peptide

Which pathogen is consistent associated with nosocomial pneumonia? A. Streptococcus pneumoniae B. Mycoplasma pneumoniae C. Haemophilus influenzae D. Pseudomonas aeruginosa

D. Pseudomonas aeruginosa

A patient is diagnosed with an antidiuretic hormone-producing lung tumor. What is the most likely diagnosis? A. Squamous cell carcinoma B. Adenocarcinoma C. Undifferentiated large cell carcinoma D. Small cell carcinoma

D. Small cell carcinoma

Which diagnosis finding is characteristically present when the parietal pleura is infected? A. Enlarged finger tips B. PaCO2≥50 mm Hg with pH ≤7.25. C. Bloody sputum D. Unique breath sound called a friction rub

D. Unique breath sound called a friction rub

Which peptide has the function of regulating sodium and water balance? A. Atrial natriuretic peptide (ANP) B. Brain natriuretic peptide (BNP) C. C-type natriuretic peptide D. Urodilantin peptide

D. Urodilantin peptide

Which structure is the source of nutrients for the blood vessels? A. Tunica intima B. Tunica media C. Adventitia D. Vasa vasorum

D. Vasa vasorum

Which statement regarding the zones of the lung is accurate? A. Zone 1 is above the level of the left atrium. B. In zone 2, alveolar pressure exceeds pulmonary arterial and venous pressures. C. Zone 2 is a small part of the lung at the apex. D. Zone 3 is in the base of the lung.

D. Zone 3 is in the base of the lung.

Which phrase is the definition of cardiac preload? A. Impedance to ejection of blood from the left ventricle B. Wall tension related to internal blood vessel radius C. A lower than normal tension curve D.Pressure generated by the end-diastolic volume

D.Pressure generated by the end-diastolic volume

Hypertonic Fluids

D5/NS D5/LR D5/0.45 NS

Isotonic Fluids

D5W Normal Saline Lactated Ringer's NS and LR are true isotonic solutions. D5W is isotonic, however, once it is infused into the blood stream, the dextrose (glucose) is quickly metabolized and used up. So the D5W becomes hypotonic when it is in the body.

When discussing hyperchloremia (elevated serum chlorine), it is correct to state that it: A: occurs with a deficit of sodium. B: arises with an excess of bicarbonate. C: has specific symptoms such as thirst. D: is a result of an underlying disorder

D: is a result of an underlying disorder

What is the synapse for Mg+

DTR-Deep Tendon Reflexes

What part of the body does Mg+ effect?

DTR-deep tendon reflexes

Medical Management of Hypercalcemia

Decreased calcium intake Increased fiber Monitor for sings of digitalis toxicity if the client is receiving digoxin

Hypocalcemia Etiology:

Decreased intake/absorption, decreased availability (hypoparathyroidism), decreased calcium excretion.

Hypovolemia

Deficiency of ECF

BUN Increases with

Dehydration Decreased renal function GI bleeding Increased protein intake Fever Sepsis

Clinical Manifestations of Hypercalcemia

Depression and lethargy Nausea, vomiting and anorexia Constipation Abdominal pain Bone pain

Hypervolemia, Medical Management of

Directed at the cause Restrict sodium and fluid Pharmacologic therapy to prevent CHF and pulmonary edema -Loop Diuretics -Thiazide Diuretics -K+ sparing Diuretics -Hemodialysis -Nutritional therapy

Hypornatremia ECF deficit (saline deficit) Manifestations: (hypovolemia)

Dizzy, tachycardia, increased pulse, BP changes (hypotension, postural hypotension), poor skin turgor (tenting over sternum), oliguria, depressed fontanels, flat neck veins, weight loss.

If the Na+ level is normal is it a ICF or a ECF problem?

ECF *volume issue

When would you see JVD?

ECF Excess

Na 140 P 96 JVD +3 edema

ECF Hypernatremia

Na 139 P 104 T 99.3 3+ edema

ECF Hypernatremia *w/ edema must be excess

Na 135 P 110 BP 110/70 Wt 158 today 160 yesterday Increased albumin/HCT

ECF Hyponatremia

Na 142 BP 154/98 supine 120/60 upright P 70 supine 90 upright increased protein

ECF Hyponatremia

Na+ level 140 is this an ICF or ECF problem? Excess or Deficit? BP 98/52 P-158

ECF- deficit

Hypervolemia, Clinical Manifestations of

Edema Distended neck veins (DNV) Dyspnea Rales Tachycardia Elevated blood pressure nWeight gain

Patients that you would not give hypotonic fluid solution

Edema, congestive heart failure, lung problems because you would push water into cells

Hypovolemia, Assessment and Diagnostic Findings

Elevated BUN in relation to the serum creatinine (a ratio greater than 20:1) Increased hematocrit Increased specific gravity Increased urine osmolality nMay have imbalances of sodium or potassium

A nurse is assessing a client who has emphysema. The nurse should report which of the following assessment findings? 1. Digital clubbing 2. Elevated temperature 3. Barrel-shaped chest 4. Diminished breath sounds

Elevated temperature Clients who have emphysema are at risk for the development of pneumonia and other respiratory infections. A nurse should report an elevated temperature to the provider, as this indicates a possible respiratory infection.

Positive Trousseau's Sign

Elicited by inflating a blood pressure cuff on the upper arm to about 20mm Hg above systolic pressure. Within 2-5 min, carpopedal spasm will occur as ischemia of the ulnar nerve develops

Hypovolemia, Medical Management

Encourage p.o fluids IV therapy -

BUN Decreases with

End stage liver disease A low protein diet Starvation Any condition that results in expanded fluid volume

Hypernatremia, Signs &Symptoms

Excessive thirst, Elevated temperature Dry, sticky mucous membranes Mental status changes: -Lethargy -Restlessness -irritability Seizures-Hypotension-Tachycardia Nausea, vomiting and anorexia noliguria

Edema of the brain, larynx, or lungs is not an acute life-threatening condition. T/F

False-

Decreased vascular volume would yield manifestations such as full and bounding pulses, venous dissension, & signs and symptoms of pulmonary edema, such as SOB, crackles, dyspnea, and cough. T/F

False- All of these are sign of increased vascular volume.

Potassium is the most abundant cation in the body. T/F

False- Sodium

The extracellular compartment contains approximately two thirds of the body water in healthy adults and is the larger of the two compartments. T/F

False-intracelular is larger

Hypovolemia

Fluid Volume Deficit 1. ECF fluid is decreased 2. Water and electrolytes are lost in the same proportion. 3. Serum electrolyte concentration remains unchanged 4. Should NOT be confused with dehydration in which water only is lost and serum sodium levels increase

Hypervolemia

Fluid Volume Excess Extracellular fluid is increased due to the abnormal retention of water and sodium in the same proportions in which they normally exist in extracellular fluid

Intercellular Fluid (ICF)

Fluid inside of cells 40% of body weight

Systemic Complications

Fluid overload Air embolism Septicemia and other infections

Regulators that Maintain Normal Fluid Balance

Food and fluid intake Kidneys Skin Lungs GI tract

manifestation of metabolic acidosis

Headache, Lethargy, nausea, vomiting, diarrhea, dry skin, warm skin, coma and death

manifestations of Hypokalemia

Heart arrhythmia, bracycardia, hypotension, irregular pulse, constipation, shallow respiration, irratibility, confusion, skeletal muscle weakness, decrease peristalis, alkalodic

Etiology of Hypovolemia

Hemorrhage, u/d 3rd spacing, decreases aldosterone

What is the synapse for Ca+

Hyper:Bones Hypo:Muscles

Clinical Manifestations of Hypomagnesemia

Hyperexcitability Muscle weakness Tremors Seizures Mood alterations - apathy, depression Cardiac arrhythmias such as PVCs, SVT, and ventricular fibrillation

If I drank too much seawater, what would I have?

Hypermagnesemia

If the underlying problem is metabolic then.....

Hyperventilation or hypo ventilation will compensate

What regulates ADH?

Hypothalamus

What regulates thirst?

Hypothalamus

If the Na+ level is abnormal is it a ICF or a ECF problem?

ICF

Na 149 BP 154/98 RR 12 Increased hemacrit + BUN restless w/ change in LOC

ICF Hypernatremia

Na 127 P 88 BP 154/98 R 16

ICF Hyponatremia

Na+ 128 is this an ICF or ECF problem? Excess or Deficit? Confusion, Weight gain,

ICF-Hyponatremia Excess

Na+ level 150 is this an ICF or ECF problem? Excess or Deficit? restless, weightless, dry mucous membrane, tachycardia, depressed fontanels

ICF-deficit Hypernatremia

Medical Management of Hypermagnesemia

IV Calcium Gluconate: antagonizes the neuromuscular effects of magnesium

Medical Management of Hypochloremia

IV Therapy Foods high in chloride Review the use of diuretics Nursing Management I & O Monitor level of consciousness Assess muscle strength

Medical Management of Hypomagnesemia

IV administration of magnesium Sulfate - 1 gm in 100ml over one hour Encourage client to eat foods irch in magnesium, such as green leafy vegetables, nuts, legumes, whole grains, and seafood.

Nursing Management of Hypophosphatemia

Identify clients at risk Monitor Signs and Symptoms Encourage dietary intake of foods such as milk, poultry and whole grains

Hyponetremia, Nursing Management

Identify patients at risk Use prevention and early intervention Monitor I&O Weigh the client daily Monitor for signs and symptoms Monitor lab results Encourage a diet high in sodium when appropriate: broth, tomato juice

Causes of Hypokalemia

Inadequate intake, Too much output Severe GI losses from Vomiting, Diarrhea , NG suction Laxative abuse, Starvation, Anorexia nervosa % bulimia, Alcoholism, Hypercalcemia, Hyperglycemia, DKA, Drugs such as: diuretics, antibiotics, corticosteriods and insulin

Nursing Management Hypocalcemia

Increased dietary calcium to 1000-1500mg/day. Foods high in calcium are milk, green leafy vegetables, canned salmon, sardines and fresh oysters. Encourage exercise Teach the patient about medications such as Fosamax, Actonel, Evista and calcitonin which reduce the rate of bone loss

Medical Management of Hypokalemia

Increased dietary potassium. bananas, apricots, oranges, raisins, melons, vegetables, legumes, whole grains, milk and meat. IV replacement therapy Potassium chloride is usually ordered The max admin rate is 10-20 mEq/hr. KCL is administered with a infusion pump!!! It is usually diluted as 10-20 mEq in 100 cc of D5W to run over one hour. Use an adequate IV line. Potassium will irritate the vein and cause a burning sensation. May be given as 20-40mEq in one liter of D5/0.45 saline for maintenance replacement. The client must have adequate urine output before administration.

Hyponatremia ICF Excess Etiologies:

Increased hypotonic fluids, psychogenic polydipsia (addicted to drinking) , too much ADH,

Hypermagnesemia Etiology:

Increased intake/absorption (excess use of Milk of Magnesia as an antacid or laxative; ingestion or aspiration of seawater), decreased excretion (renal failture)

Hypercalcemia Etiology

Increased intake/absorption, Calcium shifting from bone (hyperparathyroidism, immobilization, bone tumors), decreased calcium excretion

Hypomagnesemia Manifestations:

Increased muscle tone, hyperactive reflexes, trousseau's, chvostek's, cardiac dysrhythmias

Manifestation of Hypercalcemia

Increased neuromuscular excitability ( nerves less able to fire), bradycardia, cardiac arrest, kidney stones, bone pain, osteoporosis

Local Complications

Infiltration Phlebitis Thrombophlebitis Clotting and obstruction

Hypernatremia, Causes of

Ingestion of salt Water deprivation Dehydration IV administration of hypertonic saline solution or hypertonic tube feedings Watery diarrhea Excessive aldosterone secretion Diabetes Insipidous

Hypokalemia

K+ serum levels <3.5 mEq/L

Hyperkalemia

K+ serum levels >5 mEq/L

What electrolytes do you loose when you emesis:

K+, Mg+, Fluid, H+ (acid)

What electrolytes do you loose when you urinate:

K+, Na+, Fluid

What electrolytes do you loose when you poop:

K+, Na+, Mg+, Fluid, HCO3 (base)

What excretes K+

Kidneys

Where is renin produced?

Kidneys

Assessment & Diagnostic Findings-Hyperkalemia

Lab results - postassium level is more than 5.0 mEq/L EKG Changes: Ventricular dysrhythmias Elevated T waves ST depression Prolonged PR interval and QRS duration Absent P waves

Assessment and Diagnostic Findings of Hypokalemia

Lab results: Potassium less than 3.5 mEq/L EKG Changes Flattened T waves ST depression Prolonged PR interval Characteristic U waves Irregular pulse due to ventricular arrythmias

Causes of Hypophosphatemia

Lab results: serum level below 2.5mg/dl Causes: Malnutrition Anorexia Alcoholism Parenteral nutrition DKA Thermal burns

Hypervolemia, Assessment and Diagnostic Findings

Lab values: -Decreased BUN and hematocrit due to plasma dilution -Normal serum sodium -CXR (chest x-ray) may show pulmonary congestion

Hypernatremia, Assessment and Diagnostic Findings

Labs: -Increased serum sodium -Decreased urine sodium -Increased urine specific gravity and osmolality

Clinical Manifestations Hypokalemia

Leg cramps - most common symptom Weakness, fatigue,Nausea, vomiting, anorexia, Decreased bowel motility, constipation, Muscle weaness, Hypotension Mental status changes-confusion, lethargy Hyporeflexia, Irregular pulse Ventricular arrythmias and cardiac arrest

Extracellular Fluid (ECF)

Made up of plasma & interstitial fluid

Hypomagnesemia

Mag serum levels <1.3 mg/dl

Hypernatremia, Nursing management

Maintain I&O Obtain daily weights Monitor vital signs carefully Monitor for mental status changes

Hypovolemia, Nursing Management

Maintain patent airway Administer oxygen as ordered Perform frequent assessment of: Mental status, vital signs, Weight, breath sounds Sin color & turgor, mucous membranes, Urine output, lab results Maintain IV access with a large bore catheter. Administer fluids, blood and medications as ordered by M.D. Monitor client's response to treatment

BUN

Measures urea Normal value is 10-20mg/dl

Hypermagnesemia Pathophysiology:

Mg+ controls the amount of acetylcholine (Ach) in neuromuscular junctions. When Mg + is high, very little ACh is present, so few impulses are transmitted leading to muscle weakness

Medical Management of Hyperkalemia

Monitor EKG's, Restrict potassium rich foods IV Calcium Gluconate, IV sodium bicarbonate IV Regular insulin and hypertonic dextrose solution Kayexelate - a cation exchange resin Can be administered p.o, NGT, GT or by retention enema Action: potassium is exchanged for sodium in the intestinal tract. Potassium is then excreted in the stool May cause diarrhea

Nursing management of Hyperchloremia

Monitor I&O Monitor vital signs Monitor arterial blood gases

Nursing Management of Hyperkalemia

Monitor changes in vital signs, EKG's, lab results Assess signs and symptoms

Nursing Management-Hypokalemia

Monitor lab results-report abnormalities to the doctor Monitor for symptoms Maintain I&O Monitor vital signs Assess heart rate, rhythm and EKG

Nursing Management of Hypermagnesemia

Monitor level of consciousness Assess patellar reflexes Monitor vital signs Fluid intake

Osmosis

Movement of fluid across semi-permeable membranes from an area of low particle concentration to an area of high particle. Goal is to equalize the concentration of particles on either side of the membrane

Diffusion

Movement of particles across a semi-permeable membrane from an area of high concentration to an area of low concentration. Goal of diffusion is to equalize the concentration of particles on either side of the membrane

purpose of K+

Muscle cell resting membrane potential,* heart*, transmission and conduction of nerve impulses, affects acid/base balance, needed for Na+/K+ pumps

Hyperkalemia Pathophysiology:

Muscle cell resting membrane potentials altered: cell is hypopolarized. Once discharged, unable to contract again.

Clinical Manifestations of Hyperkalemia

Muscle weakness, cramps, Irritability, Slurred speech Anxiety, Confusion, Nausea, vomiting and diarrhea Ventricular arrhythmias, Cardiac arrest, Death

What electrolyte do you NOT loose when you emesis?

Na+

Hyponatremia

Na+ levels <135 intracellular osmolality > extracellular osmolality creating fluid to move in to cell

Hypernatremia

Na+ levels > 145 mEq/L intracellular osmolality < fluid moves out of the cell

What part of the body does Na+ effect?

Neuro

Hematocrit

Normal value 44-52% in males& 39-47% in females Measures the % of RBCs in the whole blood Level increases with Dehydration Polycythemia ( blood disorder in which your bone marrow makes too many red blood cells.)decreases with,Anemia,Overhydration

Creatinine

Normal value is 0.7-1.5mg/dl An end product of muscle metabolism Better indicator of renal function than the BUN because it does not vary with protein intake and metabolic state. Level increases when renal function decreases

Urine Specific Gravity

Normal value is 1.010-1.025 Measures the kidneys' ability to excrete or conserve water. Varies inversely with urine volume. The larger the volume of urine, the lower the specific gravity. The smaller the volume of urine, the higher the specific gravity. Is compared to the weight of distilled water, which has a specific gravity of 1.000

What are Hyperparathyroidism Manifestations?

Osteoporosis, pathologic fractures, bone pain,

If H+ is low then

PH is high (alkaline) >7.45

If H+ is high then

PH is low (acidic)<7.35

A nurse is caring for a client who has acute respiratory failure. Which of the following laboratory findings should the nurse expect? 1. arterial pH 7.50 2. paCO2 25 mm Hg 3. SaO2 92% 4. PaO2 58 mm Hg

PaO2 58 mm Hg the nurse should expect the client who has acute respiratory failure to have a lower partial pressures of oxygen

Filtration

Passage of fluid through a semi-permeable membrane

What are the intracellular Electrolytes?

Potassium (K+) Magnesium (Mg+) Phosphate (PO4) Proteins

Hypokalemia

Potassium level is less than 3.5 mEq/L Potassium is not stored in the body, so daily intake is required 80-90% is excreted by the kidneys

K+

Potassium. 3.5-5 mEq/L . Most ICF cation Regulates neuromuscular exictability and muscle contraction It is regulated primarily by the kidney. Decreased urine output equals decreased potassium excretion. There is an exchange mechanism with the sodium ion. When sodium is retained, potassium is excreted

Interstitial Fluid (IF)

Potential spaces: bowel, peritoneum, pleural space. 20% of body weight

Serum osmolality

Reflects the concentration of sodium in blood

Urine osmolality

Reflects the concentration of urea, creatinine and uric acid in urine.

Causes of Hyperkalemia

Renal failure Cellular damage Trauma burns Administration of large amounts of potassium Diabetes Acidosis Addison's Disease

Causes of Hypermagnesemia

Renal failure is most common cause Untreated DKA Adrenocortical insufficiency Addison's disease Hypthermia Excessive use of antacids or laxatives

What does RAAS stand for,what causes it, and what does it do?

Renin-Angiotensin-Aldosterone-System It is caused by decreased kidney profusion, low BP, etc. Retains Na+ & H2O, loose K+, Increased fluid, Vasoconstriction =High Blood Pressure *affects both electrolytes and fluid

The body compensates for Metabolic Alkalosis by......

Respiratory compensation is difficult-hypoventalation is limited by hypoxia usually occurs with renal dysfunction so can't count on the kidneys to help compensate.

Physiological Buffers include:

Respitory-CO2 excretion-takes minutes Renal-H+ excretion-also retain HCO3 available- takes sometimes hours 12 hourish

Hyponatremia ICF Excess manifestations:

Serum Na+ < 135, polyuria, weight gain, irritability, confusion, lethargy, seizures, coma, depression, malaise,nausea, vomiting.

Hypernatremia ICF deficit manifestations:

Serum Na+ > 145 Confusion, lethargy, seizures, oliguria, thirst.

Hyperkalemia Manifestations:

Skeletal muscle weakness, cardiac dysrhythmias, bradycardia, heart block, diarrhea, tingling, numbness.

Manifestation of acidosis

Sleepy, slow shallow breathing, disorientation, skin warm and flushed die to vasodialation caused by excessice CO2, coma, death

What are four extracellular Electrolytes?

Sodium (Na+) Bicarbonate (HCO3-) Chloride (Cl-) Calcium (Ca+)

Na+

Sodium 135-155 mEq/L. a.Most abundant electrolyte in exctracellular fluid. b.Involved in maintaining water balance, transmitting nerve impulses & contracting muscles c.It is the primary determinant of extracellular fluid osmolality

Hypernatremia

Sodium Excess Sodium level is greater than 145mEq/L Occurs when there is a greater than normal concentration of sodium in the extracellular fluid from extreme water loss or sodium excess

Hyponetremia, Medical Management

Sodium Replacement -P.O -IV lactated ringers or 0.9 sodium chloride •Administer carefully •Avoid correcting the hyponatremia too quickly as this can cause cerebral edema Water Restriction -800ml/24 hours

Hyponatremia

Sodium deficit Occurs when there is a less than normal concentration of sodium in the blood, either from sodium loss or water excess The sodium level is less than 135 mEq/L

Electrolytes

Substances that, when in solution, seperate (or dissociate) into electrically charged particles called ions

Hypernatremia ECF excess (saline excess) Manifestations: (hypervolemia)

Sudden weight gain, edema, polyuria, hypertension, bulging fontanels.

What is insensible fluid loss?

Sweat, H2O Vapor (not measurable)

A nurse is caring for a client who is taking albuterol. For which of the following adverse effects should the nurse monitor the client? 1. Hyperkalemia 2. Dyspnea 3. Tachycardia 4. Candidiasis

Tachycadia

Osmosis

The movement of fluid from an area of low solute concentration to an area of high solute concentration until the solutions are of equal concentrations

Diffusion

The movement of substances from an area of high concentration to an area of low concentration

Filtration

The movement of water and solutes from an area of high hydrostatic pressure to an area of low hydrostatic pressure

Hydrostatic pressure

The pressure of fluid against either side of the wall of the capillary. "push" of fluid

Osmotic pressure

The pull of particles contained in the fluid on either side of the semi-permeable membrane. "pull" of particles

Hyperkalemia

There is an excess amount of potassium in the extracellular fluid Potassium level is greater than 5 mEq/L

In the middle of the capillary beds

There is no net movement of water. solutes now can diffuse according their concentration gradients. solutes such as glucose, salt and minerals

Manifestation of alkalosis

Tingly sensation. Over stimulation of the central and peripheral nervous systems. carpal tunnel spasms, tetany, nervousness, tense, seizures, numbness, light headed, anxious, loss of consciousness, death

Medical Management of Hyperphosphatemia

Treat underlying disorder Avoid laxatives and enemas Low phosphorus diet: avoid hard cheese and cream Nursing Management: Monitor at risk clients

Polydipsia is a term that means excessive thirst. T/F

True

A PH of 7.30 with a CO2 of 32 and HCO3 of 20

Uncompensated metabolic acidosis

A PH of 7.23 with a CO2 of 50 and HCO3 of 29

Uncompensated respiratory acidosis

A PH of 7.55 with a CO2 of 29 and HCO3 of 20

Uncompensated respitory alkalosis

What is sensible fluid loss?

Urine, food, diarrhea, vomiting (measurable)

Extracellular imbalances are what kind of issue?

Volume *Both too high (excess) or too low (deficit)

Hypovolemia, Causes of

Vomiting Diarrhea GI suctioning Sweating Decreased fluid intake Hemorrhage Third space fluid shifts

retaining sodium causes increase BP

Water follows sodium. If Sodium is retained than water will be retained as well causing an increase in blood volume

Hyponetremia, Signs & Symptoms

Weak rapid pulse (tachycardia) Hyptension. Mental status changes -Lethargy -confusion, Seizures, Stroke, Coma, Death Muscle problems -Cramps, twitching, weakness, Dry skin

Hypovolemia, Clinical Manifestations of

Weight loss Decreased sin turgor Oliguria Postural hypotension Tachycardia Increased temperature Cool, clammy skin Anorexia Muscle weakness and cramps

Oncotic pressure

When H2O is being "pulled" by proteins . *just involved with proteins

Hypomagnesemia Pathophysiology:

When Mg2+is low, too much ACh is released into neuromuscular junction leads to muscle contraction

Hypercalcemia & kidney stones

When a patient has too much calcium in the blood, he becomes prone to kidney stones. Many kidney stones are made up of calcium,some are uric acid. If someone has high calcium levels, strain the urine

compensation is

When the body brings PH back into normal ranges Can be fully or partially

Positive Chvostek's Sign

When you tap the facial nerve 2cm in front of the earlobe and just below the zygomatic arch, the muscles supplied by the facial nerve begin to twitch.

A nurse is caring for four clients. Which of the following clients is at greatest risk for pulmonary embolism? 1. a client who is 12 hr postoperative following a total hip arthroplasty 2. a client who is 8 hr postoperative following an open surgical appendectomy 3. a client who is 2 hr postoperative following an open reduction external fixation of the right radius 4. a client who is 4 hr postoperative following a laparscopic cholecystectomy

a client who is 12 hr postoperative following a total hip arthroplasty the nurse should identify the client who has undergone a total hip replacement surgery is a greatest risk for a pulmonary embolus due to decreased mobility of the affected extremity. Therefore, this is the priority finding.

lactic acid is what kind of acid?

a fixed acid and it is harder for the body to get rid of it

if a person is not breathing what will the PH be?

a person will be Acidosis

hyperventilating will cause what to happen to PH?

a person will be alkadosis

Asthma is precipitated by which of the following inflammatory mediators? (1+ answers) a. histamine b. prostaglandins c. leukotrienes d. neutrophilic infiltration

a,b,c,d

Normal systolic pressure within the left ventricle is in the range of: a. 90 to 140 mm Hg b. 15 to 30 mm Hg c. 0 to 10 mm Hg d. none

a. 90 to 140 mm Hg

During atrial systole, the: a. atrioventricular valves are open b. atria are filling c. ventricles are emptying d. semilunar valves are open

a. atrioventricular valves are open

Bacterial infective endocarditis differs from rheumatic heart disease because of which of the following? (1+ answers) a. bacterial endocarditis is an infection of the heart, endocardium, and valves b. it always follows rheumatic fever c. it may occur following dental or bladder catheterization procedures d. it commonly involves the vena cava valve e. it is caused by a type III hypersensitivity

a. bacterial endocarditis is an infection of the heart, endocardium, and valves c. it may occur following dental or bladder catheterization procedures

In metabolic alkalosis clients have clinical manifestations of muscle cramps, spasms, hyperreflexia, & tetany caused by: a. binding of Ca+ in a alkaline environment causing hypocalemia b. release of Ca+ in an alkaline environment causing hypercalemia c. decreased H+ excretion d. high serum pH resulting of a deficit of carbonic acid due to an excess excretion or a deficit of CO2

a. binding of Ca+ in a alkaline environment causing hypocalemia

In bronchial asthma; a. bronchial muscles contract b. bronchial muscles relax c. mucous secretions decrease imbalances within the CNS develop

a. bronchial muscles contract

Chamber volume increase is observed in what type of cardiomyopathy? a. dilated b. hypertrophic c. restrictive d. a and b e. a, b, c are correct

a. dilated

Which ordinarily brings about the greatest increase in the rate of respiration? a. excess carbon dioxide b. increased CO2 c. increased arterial pH d. a sudden rise in blood pressure

a. excess carbon dioxide

Select all that apply: Prolonged diarrhea causes fluid & electrolyte embalances a. fluid volume deficit b. hyponatremia c. hypomagnesemia d.hypokalemia e.hypocalcemia

a. fluid volume deficit b. hyponatremia c. hypomagnesemia d.hypokalemia & base HCO3 *watery diarrhea you loose more H2O then Na+, resulting in a ECF issue

Compared with arteries, veins: a. have a larger diameter b. are thick-coated c. recoil quickly after distension d. a and b

a. have a larger diameter

Strong acids (1+ answer) a. include phosphoric acid b. contribute many H+ to the solution c. have a pH of 7 d. have a pH of 14 e. are eliminated by the renal tubules f. are good buffers.

a. include phosphoric acid b. contribute many H+ to the solution e. are eliminated by the renal tubules

If the sympathetic nervous system stimulation of the heart predominates over parasympathetic nervous system stimulation, the heart will: a. increase its rate b. contract with greater foce and at a slower rate c. decrease its rate and force of contraction d. contract with less force and at a higher rate

a. increase its rate

Which of the following increases the respiratory rate? a. increased PaCO2, decreased pH, decreased PaO2

a. increased PaCO2, decreased pH, decreased PaO2

T: (1+) a. is caused by an aerobic bacillus b. may affect other organs c. involves a type III hypersensitivity d. antibodies may be detected by a skin test

a. is caused by an aerobic bacillus b. may affect other organs d. antibodies may be detected by a skin test

Primary hypertension: a. is essentially idiopathic b. can be caused by renal disease c. can be caused by hormone imbalance d. results from arterial coarctation e. b, c, d, are correct

a. is essentially idiopathic

The metastasis of lung squamous cell carcinoma is a. late b. very early and widespread c. early d. early and widespread e. never seen

a. late

The Frank-Starling law of the heart concerns the relationship between: a. length of cardiac muscle fiber and the strength of contraction b. stroke volume and arterial resistance c. rapidity of nerve conduction and stroke volume d. systolic rate and cardiac output

a. length of cardiac muscle fiber and the strength of contraction

In congestive heart failure, the pump or myocardium itself fails because of which of the following?: (1+ answers) a. loss of contractile force of the heart b. hypertension c. cardiac dysrhythmias d. intermittent claudication from occlusive vascular disease

a. loss of contractile force of the heart b. hypertension c. cardiac dysrhythmias

Pulmonary emboli usually (1+) a. obstruct blood supply to lung parenchyma b. originate from thrombi in legs c. occlude pulmonary vein branches d. occlude pulmonary artery branches

a. obstruct blood supply to lung parenchyma b. originate from thrombi in legs d. occlude pulmonary artery branches

When blood exhibits a turbulent flow: a. resistance increases b. greater blood viscosity is present c. the fluids have greater velocity than with laminar flow d. hydrostatic pressure is greater than when the fluids demonstrate laminar flow

a. resistance increases

The dorsal respiratory group (DRG) of neurons: a. sets the automatic rhythm of respiration b. modifies the rhythm of respiration c. is active when increased ventilatoin is required d. none

a. sets the automatic rhythm of respiration

Oxygen diffusion from the alveolus to the alveolar capillary occurs because: a. the PaO2 is less in the capillary than in the alveolus b. the PaO2 is greater in the atmosphere than in the arterial blood c. oxygen diffuses faster than CO2 d. the PaO2 is higher in the capillary than in the alveolus

a. the PaO2 is less in the capillary than in the alveolus

When the diaphragm and external intercostal contract; a. the intrathoracic volume increases b. the intrathoracic pressure increases c. the intrathoracic volume decreases d. none

a. the intrathoracic volume increases

Select all that apply: Prolonged emesis causes fluid & electrolyte embalances a. fluid volume deficit b. hyponatremia c. hypomagnesemia d.hypokalemia e.hypocalcemia

a.fluid volume deficit b.hypomagnesemia c.hypokalemia

etiology of acidosis

accumulation of acids or loss of bases through diarrhea or renal dysfunction. increase concentration of H+.

A nurse is caring for a client who has pulmonary embolism. Which of the following interventions is the priority? 1. provide a quiet environment 2. encourage use of incentive spirometry every 1 to 2 hr 3. initiate continuous cardiac monitoring 4. administer heparin via continuous IV infusion

administer heparin via continuous IV infusion Using the airway, breathing, circulation approach to client care, the nurse should place priority on stabilizing circulation to the lungs by administering heparin to prevent further clot formation. Therefore, this is the priority intervention.

A nurse is caring for a client who has active tuberculosis. Which of the following isolation precautions should the nurse implement? 1. airborne 2. neutropenic 3. contact 4. droplet

airborne the nurse should initiate airborne precautions for the client who has tuberculosis because tuberculosis is a respiratory infection that is spread through the air

A nurse is planning care for a client who has asthma. Which of the following medications should the nurse plan to administer during an acute asthma attack? 1. cromolyn sodium 2. prednisone 3. fluticasone and salmeterol 4. albuterol

albuterol the nurse should administer albuterol, a short-acting beta 2-adrenergic agonist, as it acts quickly to produce bronchodilation during an acute asthma attack.

A nurse is caring for a client who is postoperative and develops an acute onset of severe chest pain and worsens upon inspiration. The client is anxious and tachypneic. Which of the following actions should the nurse take first? 1. Apply supplemental oxygen 2. increase the rate of IV fluids 3. administer pain medication 4. initiate heparin therapy

apply supplemental oxygen when using the airway, breathing, circulation approach to client care, the greatest risk to the client is severe hypoxemia. Therefore, the firest action the nurse should take is to apply supplemental oxygen.

A nurse is caring for a client receiving mechanical ventilation. The low pressure alarm sounds. Which of the following should the nurse recognize as a cause of the alarm? 1. excess secretions 2. kinks in the tubing 3. artificial airway cuff leak 4. biting on the endotracheal tube

artificial airway cuff leak An artificial airway cuff leak interferes with oxygenation and causes the low pressure alarm to sound.

Given that the oxygen content of blood equals 1.34mL of O2 per gram of hemoglobin times the arterial oxygen saturation percentage, if hemoglobin concentration is 15g/dL and arterial saturation is 98%, what is the arterial oxygen content? a. 13.2 mL per dL of blood b. 19.7 mL per dL of blood c. 14.7 mL per dL of blood d. none

b. 19.7 mL per dL of blood

Identify the correct sequence of the portions of the pulmonary circulation: 1 - pulmonary vein 2 - pulmonary arteries 3 - lungs 4 - right ventricle 5 - left atrium a. 1 5 3 2 4 b. 4 2 3 1 5 c. 4 1 3 2 5 d. 5 2 3 1 4 e. 5 1 3 2 4

b. 4 2 3 1 5

As HCO3- shifts from the RBC to the blood plasma, it is expected that the plasma: a. Na+ increases b. CL- shifts into RBC c. K+ increases d. pH decreases

b. CL- shifts into RBC

In the normal cardian cycle, which of the following occurs? (1+ answer) a. the right atrium and right ventricles contract simultaneously b. The two atria contract simultaneously, whereas the two ventricles relax c. The two ventricles contract simultaneously, where as the two atria relax d. Both the ventricles and atria contract simultaneously to increase cardiac output

b. The two atria contract simultaneously, whereas the two ventricles relax c. The two ventricles contract simultaneously, where as the two atria relax

A 53 y/o man was admitted to the ER after experiencing shortness of breath, weakness, cardiac dysrhythmias and chest pain that did not subside after nitroglycerin therapy. Lab tests showed that the patient had an elevation of serup CPK, troponin, and SOT or AST. ECG tracings revealed a prominent Q wave and an elevated ST segment. The most probable diagnosis is: a. transient ischemic attack b. an acute myocardial infarct c. an attack of unstable angina pectoris d. Prinzmetal angina e. coronary artery vasospasm

b. an acute myocardial infarct

The complications of uncontrolled hypertension include all of the following except: a. cerebrovascular accidents b. anemia c. renal injury d. cardiac hypertrophy e. all

b. anemia

Cardiac muscle differs from skeletal muscle in that cardiac muscle is: (1+ answer) a. arranged in parallel units b. arranged in branching networks c. multinucleated d. singly nucleated e. more accessible to sodium and potassium ions

b. arranged in branching networks d. singly nucleated e. more accessible to sodium and potassium ions

The position of the heart in the mediastinum is: a. inferior to the diaphragm and between the lungs b. between the lungs and superior to the diaphragm c. posterior to the trachea and anterior to esophagus d. posterior to the lungs and anterior to diaphragm

b. between the lungs and superior to the diaphragm

The blood pH is maintained near 7.4 by buffering systems. The sequence from the fastest-acting to the slowest-acting system is: a. lungs, kidneys, blood buffers. b. blood buffers, lungs, kidneys c. blood buffers, kidneys, lungs d. lungs, blood buffers, kidneys

b. blood buffers, lungs, kidneys

Most O2 is carried in the blood (blank), most CO2 is carried in the blood (blank) a. dissolved in plasma; associated with salt or an acid b. bound to hemoglobin; associated with bicarbonate/carbonic acid c. combined with albumin; associated with carboni acid and hemoglobin d. bound to hemoglobin; bound to albumin

b. bound to hemoglobin; associated with

Which factor might increase resistance to the flow of blood through the blood vessels? a. an increased inner radius of diameter of blood vessels b. decreased numbers of capillaries c. decreased blood viscocity d. decreased numbers of RBC

b. decreased numbers of capillaries

Individuals with only left heart failure would exhibit which of the following? (1+ answer) a. hepatomegaly b. dyspnea c. ankle swelling d. pulmonary edema e. peripheral edema

b. dyspnea d. pulmonary edema

complications of an infarcted myocardium likely could include: (1+ answers) a. emphysema b. heart failure c. endocarditis d. death e. systemic thromboembolism

b. heart failure d. death e. systemic thromboembolism

The release of ADH is not stimulated by: a. stress b. hyponatremia c. hypernatremia d. an increase in plasma osmolality e. a decrease in plasma volume

b. hyponatremia

Orthostatic hypertension is caused by all of the following except: a. increased age b. increased blood volume c. autonomic nervous system dysfunction d. bed rest e. severe varicose veins

b. increased blood volume

Of the 60% of the body weight made up of water, about 2/3 is: a. extracellular water b. intracellular water c. intravascular water d. interstitial water 3. none

b. intracellular water

Which is inconsistent with pneumonia? a. chest pain, cough, rales b. involves only interstitial lung tissue c. may be caused by mycoplasmas d. can be lobar pneumonia or bronchopneumonia

b. involves only interstitial lung tissue

Chronic bronchitis (1+ answer) a. is caused by lack of surfactant b. is caused by air pollutants c. exhibits a productive cough d. causes collapsed alveoli

b. is caused by air pollutants c. exhibits a productive cough

Which statement is true regarding rheumatic heart disease? (1+ answers) a. it is caused by staphylococcal infections b. it is caused by hypersensitivity/immunity to streptococci c. it damages the tricuspid valve most often d. it usually damages the mitral valve

b. it is caused by hypersensitivity/immunity to streptococci d. it usually damages the mitral valve

An elevated anion gap is associated with an accumulation of: a. chloride anions b. lactate anion c. Both a and b d. neither

b. lactate anion

The hepatic vein carries blood from the: a. vena cava to the liver b. liver to the vena cava c. aorta to the liver d. liver to the aorta

b. liver to the vena cava

Which of the following statements is true? a. lymphatic walls consist of multiple layers of flattened endothelial cells b. lymph from the entire body, except for the upper right quadrant, eventually drains into the thoracic duct c. the thoracic duct has approx the same diameter as the great veins d. lymph contains more proteins than does blood plasma e. the lymphatic system, like the circulatory system, is a closed circuit

b. lymph from the entire body, except for the upper right quadrant, eventually drains into the thoracic duct

Oxygenated blood flows through the: a. superior vena cava b. pulmonary veins c. pulmonary arteries d. coronary veins e. none

b. pulmonary veins

A lung cancer characterized by many anaplastic figures are the production of hormones is most likely a. squamous cell carcinoma b. small cell carcinomoa c. large cell carcinoma d. adenocarcinoma e. bronchial adenoma

b. small cell carcinomoa

Aldosterone controls ECF volume by: a. carbohydrate, fat and protein catabolism b. sodium reabsorption c. potassium reabsorption d. water reabsorption e. Both b and d are correct

b. sodium reabsorption

Depolarization of cardiac muscle cells occurs because of: a. the cell's interior becoming more negatively charged. b. the cell's interior becoming less negatively charged c. impermeability of the cell membrane to sodium d. impermeability of the cell membrane to potassium

b. the cell's interior becoming less negatively charged

The QRS complex of the ECG represents: a. atrial depolarization b. ventricular depolarization c. atrial contraction d. ventricular repolarization e. atrial repolarization

b. ventricular depolarization

Manifestation of Hypomagnesia

behavioral changes, irritability, increased reflexes, muscle cramps, Ataxia, Nstagmus, tetany

A nurse is caring for a client who has lung cancer. Which of the following assessment findings should the nurse expect? 1. blood-tinged sputum 2. decreased tactile fremitus 3. resonance with percussion 4. peripheral edema

blood-tinged sputum the nurse should expect blood-tinged sputum secondary to bleeding from the tumor

G.P., 50 y/o man, was referred for evaluation of BP. If he has a high diastolic blood pressure, which of the following is his reading? a. 140/82 mmHg b. 160/72 mmHg c. 130/95 mmHg d. 95/68 mmHg e. 140/72 mmHg

c. 130/95 mmHg

A 76 y/o man came to the ER after experiencing chest pain while shoveling snow. Lab tests revealed essentially normal blood levels of SGOT, CPK and LDH enzymes. The chest pain was relieved following rest and nitroglycerin therapy. The most probable diagnosis is: a. myocardial infarct b. emphysema c. angina pectoris d. hepatic cirrhosis e. acute pancreatitis

c. angina pectoris

During expiration, which relationship is true? a. as the lung volume decreases, the number of gas molecules increases b. as the lung pressure increases, the number of gas molecules increases c. as the lung volume decreases, the pressure increases d. as the partial pressure increases, less gas will dissolve in a liquid

c. as the lung volume decreases, the pressure increases

Which do(es) not significantly affect heart rate? a. sympathetic nerves b. parasympathetic nerves c. atrioventricular valves d. acetylcholine

c. atrioventricular valves

Which would not shift the blood pH toward alkalosis? a. hydrogen ion secretion in urine b. exhalation of carbon dioxide c. bicarbonate ion secretion into urine d. all of the above e. none of the above

c. bicarbonate ion secretion into urine

Hyperkalemia results after a crush injury because: a. K+ is exchanged for H+ inside the cell b. hyperaldosteronism c. cell membrane damage d. hyperglycemia

c. cell membrane damage

A milliequivalent is a unit of a. mass b. physical activity c. chemical activity d. osmotic concentration

c. chemical activity

High altitudes may produce hypoxemia by; a. right to left shunts b. hypoventilation c. decreased oxygen inspiration d. diffusion abnormalities e. all

c. decreased oxygen inspiration

Which statement is incorrect concerning hypertension? a. malignant hypertension is characterized by a diastolic pressure higher than 140 mmHg b. more than 90% of cases are of the essential or primary type. c. headache is the most reliable symptom d. when it is left untreated, the major risks include CVA's and cardiac hypertrophy

c. headache is the most reliable symptom

Laboratory studies of an adult reveal the following: Plasma sodium = 110mEq/L Plasma chloride = 85mEq/L Plasma potassium = 4.8mEq/L Plasma calcium = 5.2mEq/L Plasma bicarbonate = 26mEq/L The most likely alteration is: a. base bicarbonate deficit (metabolic acidosis) b. hypokalemia c. hyponatremia d. base bicarbonate excess )metabolic alkalosis) e. calcium deficit

c. hyponatremia e. calcium deficit

A shift to the right in the oxyhemoglobin dissociation curve: a. prevents oxygen release at the cellular level b. causes oxygen to bind more tightly to hemoglobin c. improves oxygen release and increases oxygen movement into the cells d. both a and b e. none

c. improves oxygen release and increases oxygen movement into the cells

Transmural myocardial infarction: a. displays non-STEMI b. occurs when infarction is limited to part of the heart wall c. is categorized as STEMI d. displays I-wave inversion

c. is categorized as STEMI

Blood pressure is measured as the: a. pressure exerted on ventricular walls during systole b. pressure exerted by the blood on the wall of any blood vessel c. pressure exerted on arteries by blood d. product of the stroke volume times heart rate

c. pressure exerted on arteries by blood

Backflow of blood from the arteries into the relaxing ventricles is prevented by the: a. venous valves b. pericardial fluid c. semilunar valves d. atrioventricular valves

c. semilunar valves

As the terminal bronchioles are approached: a. the epithelium becomes thicker b. mucus-producing glands increase c. the epithelium becomes thinner d. cartilaginous support increases e. the smooth muscle layer thickens

c. the epithelium becomes thinner

The pericardial space is found between the a. myocardium and parietal pericardium b. endocardium and visceral pericardium c. visceral pericardium and parietal pericardium d. visceral pericardium and epicardial pericardium

c. visceral pericardium and parietal pericardium

How do kidneys effect acid/case balance?

can eliminate large amounts of acid, and can also excrete base, can conserve and produce bicarb ions Most effective regulator of PH. last to show up

The pressure that pulls water back into the capillary

capillary colloidal osmotic pressure/oncotic pressure made from the protein particles albumin

The pressure that pushes the water from the capillary into the interstitial space

capillary filtration pressure/hydrostatic pressure

purpose of magnesium

cofactor in metabolic rxn, smooth and cardiac muscle rxns, deep tendon reflexes, ATP rxns, interferes with acetacholine

Metabolic Alkalosis

concentration of Bicarb is >26 mEq/L. Occurs because of loss of acid most common with vomiting. Excessive use of alkaline drugs, hypokalemia, certain diuretics, endocrine disorders, heavy ingestion of antacids, severe dehydration.

manifestations of Hyponatremia

confusion, irritability, lethargy, nausea, vomiting, seizures, coma *effects on the brain*

A nurse is caring for a client following the insertion of a chest tube. The nurse should plan to have which of the following items in the client's room? 1. extra drainage system 2. suture removal set 3. container of sterile water 4. nonadherent pads

container of sterile water The nurse should plan to place the open end of the tubing if it becomes disconnected into the sterile water to prevent a pneumothorax. The tubing and sterile water are then placed below the client's chest.

I dentify the normal sequence of an electrical impulse through the heart's conduction system 1 - atrioventricular bundle 2 - AV node 3 - Purkinje fibers 4 - SA node 5 - right and left bundle branches a. 4 1 2 5 3 b. 4 2 5 1 3 c. 2 4 1 5 3 d. 4 2 1 5 3

d. 4 2 1 5 3

The normal heartbeat is initiated by the: a. coronary sinus b. atrioventricular bundle c. right ventricle d. SA node e. AV node

d. SA node

Which statement is true? a. the number of ions and anions in the body must be equal. b. Intravascular molecules of protein are without charge c. The sodium ions must be united with chloride ions d. The positive and negative charges in blood plasma must be equal to each other

d. The positive and negative charges in blood plasma must be equal to each other

An acid is: a. anion b. cation c. a substance/chemical that combines with a hydrogen ion to lower pH. d. a substance/chemical that donates a hydrogen ion or a proton to the solution.

d. a substance/chemical that donates a hydrogen ion or a proton to the solution.

Localized outpouching of a vessel wall or heart chamber is: a. a thrombus b. an embolus c. a thromboembolous d. an aneurysm e. a vegetation

d. an aneurysm

Adiponectin is: a. an enzyme b. increased in obesity c. inflammatory d. antiatherogenic

d. antiatherogenic

In which sequence does PaO2 progressively decrease? a. blood in aorta, atmospheric air, body tissues b. body tissues, arterial blood, alveolar air c. body tissues, alveolar air, arterial blood d. atmospheric air, aortic blood, body tissues

d. atmospheric air, aortic blood, body tissues

Pulmonary HTN a. occurs when left arterial pressure is elevated b. involves deep vein thrombosis c. shows right ventricular hypertrophy on an electrocardiogram d. both a and c e. a, b, c

d. both a and c

Type II pneumocyte damage causes: a. increased alveolocapillary permeability b. chemotaxis for neutrophils c. exudation of fluid from capillaries into interstitium d. decreased surfactant production e. all of the above

d. decreased surfactant production

Adrenomedullin (ADM): a. exhibits powerful vasoconstriction activity b. is present only in cardiovascular tissue c. mediates sodium reabsorption d. exhibits powerful vasodilatory activity

d. exhibits powerful vasodilatory activity

An individual suffers from weakness, dizziness, irritability and intestinal cramps. Lab studies reveal: Plasma sodium = 138 mEq/L Plasma potassium = 6.8mEq/L Blood pH= 7.38 Plasma bicarbonate = 25mEq/L An EKG with tall, peaked T wave, but otherwise normal The individual is suffering from: a. hypernatremia b. hyponatremia c. hypercalcemia d. hyperkalemia e. hypokalemia

d. hyperkalemia

Shock is a comlex pathophysiologic process involving all of the following except: a. decreased blood perfusion in kidneys b. acidosis c. rapid heart rate d. hypertension e. anaerobic glycolysis

d. hypertension

The left bronchus: a. is shorter and wider than the right b. is symmetrical with the right c. has a course more vertical than that on the right d. is more angled than the right e. has more bronchial wall layers than the right

d. is more angled than the right

Which of the following would result in hyperkalemia? a. hyperaldosteronism b. h+ ions return to the blood & K+ ions return to the cell. c. metabolic alkolosis d. metabolic acidosis

d. metabolic acidosis

Sodium is responsible for: a. ICF osmotic balance b. ECF osmotic balance c. TBW osmolality d. osmotic equilibrium

d. osmotic equilibrium

The cilia of the bronchial wall: a. ingest bacteria b. trigger the sneeze reflex c. trap and remove bacteria d. propel mucus and trapped bacteria toward the oropharynx e. both a and c are correct

d. propel mucus and trapped bacteria toward the oropharynx

The respiratory unit consists of: a. cilia b. bronchiolar arteries and veins c. goblet cells and alveoli d. respiratory bronchioles and alveoli e. all

d. respiratory bronchioles and alveoli

Considering the sequence of structures through which air enters the pulmonary system, the pharynx is to the trachea as the: a. bronchioles are to the segmental bronchi b. alveoli are to the alveolar ducts c. alveolar ducts are to the respiratory bronchioles d. respiratory bronchioles are to the alveolar ducts e. all of the above

d. respiratory bronchioles are to the alveolar ducts

Alveoli are well suited for diffusion of respiratory gases because: a. they are small and, thus, have a small total surface area b. vascularization is minimal, allowing greater air circulation c. they contain four thick layers, preventing air leakage d. they contain surfactant, which helps prevent alveolar collapse

d. they contain surfactant, which helps prevent alveolar collapse

Causes of serum sodium of 150 a. chronic renal failure b. vomiting c. hyperaldosteronism d. tube feeds

d. tube feeding *hypervolemia problem w/ hyperaldosteronism

Etiology of Hypernateremia

decrease of ADH, increase Na+ intake, interal feedings, decrease thirst drive, vomiting/sux, watery stool, extreme dehydration

Etiology of hyperphosphotemia

decreased excretion, vitamin D toxicity because of increased absorption, hypoparathyrodism, kidney failure

Etiology of hypophosphotemia

decreased intake, ETOH, excessive antacid use

Hypokalemia Etiology:

decreased intake, increased excretion (hyperaldosteronism), Electrolyte shifts during alkalosis, Abnormal loss through emesis, gastric suction

Hypomagnesemia Etiology:

decreased intake/absorption (chronic ETOH), increased excretion (diabetic ketoacidosis, diuretic therapy), abnormal losses (emesis, gastric suction, fistula drainage).

Hypermagnesemia Manifestations:

decreased muscle tone, respiratory depression, bradycardia, dysrhythmias, hypotension, Depressed deep tendon reflexes, lethargy.

Chronic respiratory acidosis can be due to

depression of respiratory center in the brain due to head trauma, drug use, emphysema, paralysis of chest muscle as seen in quadriplegics, cerebral palsy

principle effect of acidosis is.......

depression of the CNS through the synaptic transmission. *messes with neurons*

The pH of saliva is about 7 and the pH of gastric juice is about 2. How many times more concentrated is they hydrogen ion in gastric juice than in saliva. a. 5 b. 50 c. 100 d. 10,000 e. 100,000

e. 100,000

One cardiac cycle: a. has a duration that changes if the HR changes b. usually requires less than 1 second to complete c. is equal to stroke volume x HR d. pumps approx 5 L of blood e. a and b

e. a and b

Which is a possible cause of varicose veins? a. gravitational forces on blood b. long periods of standing c. trauma to the saphenous veins d. both b and c are correct e. a, b, and c

e. a, b, and c

Alveoli are excellent gas exchange units because of: a. their large surface area b. a very thin epithelial layer c. extensive vascularization d. both b and c e. a, b, c

e. a, b, c

Cor pulmonale a. occurs i response to long standing pulmonary HTN b. is right heart failure c. is manifested by altered tricuspid and pulmonic valve sounds d. both b and c e. a, b, c

e. a, b, c

In ARDS, increased alveolocapillary membrane permeability is caused by: a. platelet activating factor (PAF) b. oxygen free radicals c. tumor necrosis factor (TNF) d. both a and c e. a, b, c

e. a, b, c

Pulmonary edema may be caused by abnormal: a. capillary hydrostatic pressure b. capillary oncotic pressure c. capillary permeability d. both a and c e. a, b, c correct

e. a, b, c correct

SIRS begins with an infection that progresses to: a. bactermia b. sepsis c. septic shock d. MODS e. all of the above

e. all of the above

Stretch receptors: a. are sensitive to volume changes in the lung b. are located in airway smooth muscles c. decrease ventilatory rate when stimulated d. prevent lung overinflation when stimulated e. all of the above

e. all of the above

Surfactant: a. facilitates O2 exchange b. produces nutrients for the alveoli c. permits air exchange between alveolar ducts d. facilitates alveolar expansion during inspiration e. all of the above

e. all of the above

The major muscles of inspitation is/are the: a. diaphragm b. sternocleidomastoid c. external intercostals d. internal intercostals e. both a and c are correct

e. both a and c are correct

In pericardial effusion: a. fibrotic lesions obliterate the pericardial cavity b. there is associated RA c. .tamponade compresses the right heart before affecting other structures d. arterial blood pressure during expiration exceeds that during inspiration e. c and d are correct

e. c and d are correct

Events in the dev't of atheroscleroticf plaque include all of the following except; a. accumulation of LDL cholesterol b. smooth muscle proliferation c. calcification d. decreased elasticity e. complement activation

e. complement activation

A person having a heart rate of 100 bpm, a systolic blood pressure of 200 mm Hg, and a stroke volume of 40 mL would have an average cardiac output of: a. 0.5 mL/min b. 5 L/min c. 4 mL/min d. 8000 mL/min e. none

e. none

In emphysema: a. there is increased area for gaseous exchange b. there are prolonged inspirations c. the bronchioles are primarily involved d. diaphragm movement is increased e. none

e. none

A young female became agitated and apprehensive, and she eventually lost consciousness. At the hospital ER, the following lab values were obtained. plasma sodium = 137 plasma potassium = 5.0 mEq/L Blood pH = 7.53 Serum CO2 = 22 mm Hg Plasma bicarbonate = 24 mEq/L Her immediate diagnosis was: a. hypokalemia b. metabolic acidosis c. metabolic alkalosis d. respiratory acidosis e. respiratory alkalosis

e. respiratory alkalosis

Aquaporins is/are: a. a mechanism to enable the body to adapt to hyperkalemia b. restrictive to water intake c. a mechanism that facilitates renal excretion of potassium d. antibodies known to cause hypokalemia e. water channel proteins that provide cell membrane permeability to water

e. water channel proteins that provide cell membrane permeability to water

manifestations of hypervolemia

edema, polyuria, high BP, Jugular vein distention, bulging fonts in infants, weight gain, bounding pulse

Hypervolemia

excess of ECF

How do fluids move?

fluids move through osmosis

A nurse is preparing a client for discharge following a bronchoscopy with the use of moderate (conscious) sedation. Which of the following assessments by the nurse is the priority? 1. gag reflex 2. pain level 3. dehydration 4. redness at the IV insertion site

gag reflex the greatest risk to the client is aspiration due to the depressed gag reflex. Therefore, the priority assessment by the nurse is to determine the return of the gag relfex

hypertonic solution

has a greater osmolality than the interstitial fluid. Cells shrink when in this solution

when a body is acidotic K+ levels are

high

A nurse is positioning a client who has emphysema to promote effective breathing. The nurse should place the client in which of the following positions? 1. lateral position with a pillow over the chest to support the arm 2. high-fowler's position with arms supported on the overbed table 3. semi-fowler's position with pillows supporting both arms 4. supine position with the head of the bed elevated 15 degrees

high fowlers position with arms supported on the overbed table the nurse should place the client in a position that allows for greater expansion of the chest, such as sitting upright and leaning slightly forward while supporting both arms on the overbed table

PH can effect

hormones

Etiology of hypervolemia

hyperaldosterone, glucocorticoids, renal failure, CHF, excessive IVF

Manifestations hypermagnesium

hypoactive reflexes (floppy), lethargy, drowsy, respiration paralysis

where does water transfer?

in the capillary

Etiology of hypocalcemia

inadequate intake or absorption, decrease production of parathyroid hormone (hypoparathyroidism), renal failure results in decrease activation of vitamin D, high phosphate level

Etiology of Hyponatremia

increase ADH, SIADH, excessive hypotonic IVF, Phsycogenic polydipsia, diuretics, salt wasting renal disease, vomiting, GI sux, burns due to massive fluid shifts

A nurse is caring for a client who has acute respiratory distress syndrome. Which of the following assessment findings indicates a decline in the client's condition? 1. increase in respiratory rate 2. increase in oxygen saturation 3. decrease in carbon dioxide retention 4. decrease in intercostal retractions

increase in respiratory rate an increase in respiratory rate indicates increased work of breathing and the need for improvement in oxygen delivery.

treatment of metabolic acidosis

increase ventilation, renal excretion of H+ ions if possible, K+ exchange with excess H+ in ECF (H+ into cells K+ out of cells), glucose/insulin can be used to push H+ out, IV lactate solution to increase amount of Bicarb available.

Hyperkalemia Etiology:

increased K > 5.0, Crush injury (Potassium shift from intracellular to extracellular), decreased K+ excretion (renal failure). *Crush injury: cellular rupture and massive release of potassium Potassium causes a imbalance in electronegativity and, in essence, causes the muscles to seize up due to lack of charge. battery is shorted out and dead

Etiology of Hypercalcemia

increased intake, hyperparathyroidism, decreases in vit D, decreases excretion, immobilization, multiple myeloma, lithium medication, neoplasm-breast cancer, lung cancer

manifestation of Hyperkalemia

increased muscular irritability, tingling of lips and fingers, restlessness, GI cramping, diahrrea, irritability & anxiety, most severe affects are on the heart, decrease membrane excitability, changes in ECG, decreased BP, irregular heart rhythm, bradycardia to cardiac arrest

fluid amount can be influenced by age

infants body weight is 80% water

Causes of Edema

inflammation that results in increased capillary permiability fluids go out, obstruction of lymph flow, decreased osmotic pressure-fluids are leaking out, decrease production of plasma proteins, increase of hydrostatic pressure-heart faliure, kidney disease

increase serum osmolarity

insufficient fluid volume/ dehydration

The Body compensates for respiratory alkalosis by....

kidneys conserve H+ ions and secrete Bicarb, urine becomes alkaline

The body compensates for respiratory acidosis by.......

kidneys will conserve Bicarb, Urine will become acidotic eliminate H+ ions in acidic urine, lactate will be converted to Bicarb ions in the liver.

Assessment and Diagnostic Findings of Hypocalcemia

less than 8.5. My cause: Osteoporosis, Primary hypoparathyroidism Pancreatitis, Renal failure Inadequate Vitamin D consumption

CO2 levels during respiratory alkalosis

less then 35 mm Hg *hypocapnea*

Hypercalcemia

level is greater than 10.5 Causes: Malignancies Hyperparathyroidism Immobility after severe or multiple fractures or spinal cord injury

Acids are normally produced by metabolism of what?

lipids and proteins

etiology of alkalosis

loss of acids hyperventilation, or kidneys aren't functioning. Accumulation of bases. decrease concentration of H+

Manifestation of Hypovolemia

low BP, oliguria, poor turgor, tachycardia, dizzy, flat neck veins, depressed fontanels, weight loss, concentrated blood values

Hypokalemia Pathophysiology:

low serum K+ < 3.5, hyperpolarized resting membrane potential, so cell is not responsive to stimulus, more difficult to depolarize

hypotonic solution

lower osmolality than Interstitial fluid. Cells swell and can burst in this solution

osmolarity

measured number of dissolved particles per unit of water and serum.

A nurse is caring for a client who is in respiratory distress. Which of the following devices should the nurse use to provide the highest level of oxygen via a low-flow system? 1. Nasal cannula 2. Nonrebreather mask 3. Simple face mask 4. partial rebreather mask

nonrebreather mask a nonrebreather mask is made up of a reservoir bag from which the client obtains the oxygen, a one-way valve to prevent exhaled air from entering the reservoir bag, and exhalation ports with flaps that prevent room air from entering the mask. This delivers greater than 90% FIO2, which provides the highest level of oxygen.

Generalized Edema

occurs everywhere. related to an over encompassing type of reaction or long term disease process (such as lack of protein) liver cirrhosis, congestive heart failure,

A nurse is caring for a client who is 4 hr postoperative following a total laryngectomy for laryngeal cancer. Which of the following assessments is the priority? 1. bleeding at the surgical site 2. oxygen saturation 3. urinary retention 4. level of consiousness

oxygen saturation Using the airway, breathing, circulation approach to client care, the nurse should identify the client's oxygen saturation is the priority assessment. A client who is postoperative following a total laryngectomy is at risk for hypoxia due to airway obstruction and decreased oxygen saturation is an indication of obstructed airway

A nurse is caring for a client who is postoperative and is hypoventilating secondary to general anesthesia effects and incisional pain. Which of the following ABG values support the nurse's suspicion of respiratory acidosis? 1. pH 7.50, PO2 95mm Hg, PaCO2 25 mm Hg, HCO3- 22mEq/l 2. pH 7.50, PO2 87 mm Hg, PaCO2 35 mm Hg, HCO3- 30 mEq/l 3. pH 7.30, PO2 90 mm Hg, PaCO2 35 mm Hg, HCO3- 20 mEq/l 4. pH 7.30, PO2 80 mmHg, PaCO2 55 mm Hg, HCO3- 22 mEq/l

pH 7.30, PO2 80 mmHg, PaCO2 55 mm Hg, HCO3- 22 mEq/l these ABG values indicate respiratory acidosis. The pH is less than 7.35 and the paCO2 is greater than 45 mmHG, which indicates respiratory acidosis

Second line of defense against PH change in the body

physiological buffer system

hydrostatic pressure/ blood pressure

pressure coming in from the arteriole end of the capillary pushes fluid out

oncotic pressure

pressure pulls

A nurse is caring for a client who has COPD. Which of the following findings should the nurse report to the provider? 1. oxygen saturation 89% 2. productive cough with green sputum 3. clubbing of fingers 4. pursed-lip breathing with exertion

productive cough with green sputum A nurse should report a productive cough with green sputum to the provider as this indicates an infection.

A nurse is planning care for a client who has COPD. Which of the following interventions should the nurse include in the plan of care? 1. schedule respiratory treatments following meals 2. have the client sit in a chair for 2-hr periods three times per day 3. provide a diet that is high in calories and protein 4. combine activities to allow for longer periods between activities

provide a diet that is high in calories and protein the nurse should provide a client who has COPD with a diet that is high in calories and protein and low in carbohydrates.

purpose of Chloride anion

provides electroneutrality

acute respiratory acidosis can be due to

pulmonary edema from CHF, significant trauma, pneumunothorax, asthma, COPD

Etiology of Hyperkalemia

rare, but can be caused by renal failure, addison, break down of muscle tissue damage, increased intake or excessive use of K+ suppements, shift from K+ in the ICF to ECF, diabetic ketoacidosis, ETOH, crush & burn injuries

purpose of Na+ cation

regulates the osmotic forces neuromuscular irritability, acid base balance, and cellular chemical rxn, and membrane transport

Etiology of hypermagnesium

renal insufficiency, excessive intake of magnesium containing antacids, adrenal insuffeciency

manifestation for metabolic alkalosis

respiration would be slow and shallow, hyperactive reflexes, depletion of electrolytes, atrial tachycardias, dysrhythmias that can be fatal. heart rate over 110.

Treatment for respiratory acidosis

restore ventilation, IV lactate solution, * Treat the underlying dysfunction or disease *

Isotonic fluid

same osmolality as ICF

Hyperchloremia

serum level > 106 mEq/L Clinical Manifestations: Tachypnea, Weakness, lethargy Causes: loss of bicarbonate ions via the kidney or the GI tract

Hypophosphatemia

serum level below 2.5mg/dl Clinical Manifestations: Irritability, Apprehension, Fatigue, Weakness Numbness, Confusion, Seizure, coma

Hypokalemia Manifestation:

smooth muscle weakness (respiratory arrest), weak/irregular pulse, postural hypotension, constipation, lethargy, confusion, paresthesias. *hypo no go

A nurse is caring for a client who has bacterial pneumonia. The nurse should expect which of the following assessment findings? 1. decreased fremitus 2. SaO2 95% on room air 3. temperature 38.8C (101.8F) 4. bradypnea

temperature 38.8C (101.8F) An elevated temperature is an expected finding for a client who has bacterial pneumonia.

If the lungs are damaged then......

the kidneys will compensate

Third spacing

the loss or trapping of ECF in the transcellular space such as the pericardial sac, the peritoneal cavity or the pleural cavity

manifestations of Hypernatremia

thirst, confusion, lethargy, oliguria, dry mucous membranes, *effects on the brain*

A nurse working in the emergency department is caring for a client following a chest trauma. Which of the following finders indicates a tension pneumothorax? 1. collapsed neck veins on the affected side 2. collapsed neck veins on the unaffected side 3. tracheal deviation to the affected side 4. tracheal deviation to the unaffected side

tracheal deviation to the unaffected side a tension pneumothorax results from free air filling the chest cavity, causing the lung to collapse and forcing the trachea to deviate to the unaffected side.

A nurse is caring for a client who is 1hr postoperative following a thoracentesis. Which of the following is the priority assessment finding? 1. pallor 2. insertion site pain 3. tracheal deviation to the unaffected side 4. temperature 37.3C (99.1F)

tracheal deviation to the unaffected side when using the airway, breathing, circulation approach to client care, the nurse should identify tracheal deviation as the priority assessment because this indicates a tension pneumothorax, which is a medical emergency

patients requiring rehydration with isotonic solution

turgor, low urine output, BP decrease, muscle cramps, loss of water weight

A PH of 7.12 with a CO2 of 28 and HCO3 of 11

uncompensated metabolic acidosis

A nurse is discharging a client who has pulmonary tuberculosis and is to start therapy with rifampin. The nurse should plan to include which of the following in the client's discharge teaching plan? 1. ringing in the ears is expected 2. purified protein derivative (PPD) skin test results will improve in 4 months 3. urine and other secretions will be orange. 4. take the medication with meals.

urine and other secretions will be orange rifampin will turn urine and other secretions orange.

A nurse is caring for a client who is in respiratory distress and requires endotracheal suctioning. Which of the following actions should the nurse take? 1. use clean technique to suction the client's endotracheal tube 2. use a rotating motion to remove the suction catheter 3. suction the oropharngeal cavity prior to suctioning the endotracheal tube 4. suction the client's endotracheal tube every 2 hr

use a rotating motion to remove the suction catheter the nurse should rotate the suction catheter during withdrawal to reduce the risk of tissue trauma

low serum osmolarity

usually due to over hydration or edema

A nurse is assisting with a thoracentesis. Which of the following actions is appropriate for the nurse to take when assisting with this procedure? (select all that apply) 1. wear goggles and make during the procedure 2. cleanse the area with an antiseptic solution 3. instruct the client to take deep breaths during insertion of the needle 4. position the client laterally on the affected side 5. apply pressure to the site after the needle is withdrawn

wear goggles and make during the procedure----the nurse and provider should wear these PPE items to reduce the risk of exposure to pleural fluid cleanse the area with an antiseptic solution--the use of antiseptic solution decreases the risk of infection, which is increased due to the invasive procedure apply pressure to the site after the needle is withdrawn--the application of pressure decreases the risk of bleeding at needle insertion site.

Hypotonic Solutions

•Used to treat clients who have normal blood pressure (normotensive) •Provide both electrolytes and water •Examples are 0.45% Sodium Chloride

Isotonic solutions

•Used to treat hypotension because they expand the volume of plasma •Examples are: Lactated Ringers •0.9% Sodium Chloride


Related study sets

Campbell Biology- Chapter 24 Early Life and the Diversification of Prokaryotes

View Set

Microeconomics - midterm 2 multiple choice

View Set